Free IELTS lessons signup

home

  • Academic practice
  • General practice
  • Task 1 Academic
  • Task 1 General
  • Task 2 (essay)

IELTS agree or disagree essay - band 9 guide

This is a band 9 guide to writing agree/disagree essays in IELTS Writing. Agree or disagree essay questions are very common for IELTS Writing task 2 . This type of questions asks you to say whether you agree or disagree with a given statement and justify your opinion .

In this lesson you will see IELTS writing task 2 sample question + model answer and learn

  • how to choose an opinion for agree/disagree question
  • how to generate ideas
  • how to give a band 9 answer for agree/disagree question

IELTS agree/disagree question sample

Let’s look at an example of IELTS writing task 2:

Big salary is much more important than job satisfaction.

Do you agree or disagree?

Provide relevant examples if necessary.

This is a classic example of agree or disagree question that you may get on IELTS Writing task 2.

You can watch our video tutorial on how to tackle agree/disagree questions in IELTS Writing:

Choose your opinion & generate ideas

IELTS agree or disagree essay

Agree or disagree question asks you to clearly determine whether you agree or disagree with the statement. Unlike questions that ask you to what extent do you agree or disagree , this question asks you to have a super-clear opinion. After you’ve decided your opinion, generate 2-3 supporting points for it .

For the task above you have two possible options:

  • You fully agree that big salary is more important than job satisfaction
  • You completely disagree that big salary is more important than job satisfaction

 Now let’s generate supporting points for each of the opinions:

supporting points for IELTS essay

  • Big salary is more important
  • Having a job with a high salary makes people feel satisfied no matter what kind of job they do
  • Money is essential for survival and good living
  • Job satisfaction is more important
  • Job satisfaction gives you a sense of fulfillment
  • Doing what you like keeps you motivated and therefore leads to a career growth
  • Money can’t buy happiness and it’s more pleasant to pursue what you’re interested in

For our essay, we’ll choose the second opinion.

Band 9 answer structure

After you’ve decided whether you agree or disagree and generated your supporting points, it’s time to start writing your essay.

There are a lot of ways to structure your essay, but we’ll use a structure that has been approved by many IELTS examiners to be high-scoring and coherent.

Band-9 essay structure :

Introduction

Body paragraph 1 - the 1st reason you agree/disagree

Body paragraph 2 - the 2nd reason you agree/disagree

Let’s take a look at each of these sections in detail.

Write your introduction in two sentences:

It is often argued that it is more advantageous to choose a job with high wage, even if it doesn't appeal to you at all.

I completely disagree with this opinion and think that job satisfaction is much more important than salary.

  • Sentence 1 - state the first reason you agree/disagree .

First of all, I believe that job satisfaction gives people a sense of fulfillment that no money can guarantee.

To explain the reason effectively, you can imagine that your examiner has no knowledge of this subject at all and you have to explain every detail:

Even if someone is earning a high salary, but feels tensed and compromises with his conscience, this person won’t enjoy his life. While pursuing one’s interests will always bring pleasure and feeling of satisfaction.

For example, a lot of famous researchers made their career choices not because of appealing wages, but because they were passionate about science.

That’s why it’s more important to choose the kind of work that makes you happy than to look only at a high salary.

Secondly, doing what you like keeps you motivated and therefore leads to a career growth.

In other words, there is a strong relation between job satisfaction and productivity. People who love their jobs can easily excel in their fields of work and achieve better results than those, who put salary on the first place.

  • Sentence 4 - support your idea with an example :

For instance, Henry Miller decided to leave his everyday job despite a good wage and ventured to become a writer. And after enduring years of ups and downs he became one of the most famous and well-paid authors of the twentieth century.

Thus, advantages of jobs that keep you satisfied outweigh the drawback of a low salary in a long-term perspective.

You can write the conclusion in one sentence that summarizes your opinion + 2 reasons for it :

To conclude, I strongly believe that job satisfaction is more beneficial than high salary because it makes people happy and motivated.

DO NOT write new ideas in the conclusion!

IELTS agree/disagree model answer

This is a full band-9 answer for IELTS agree or disagree question above:

It is often argued that it is more advantageous to choose a job with high wage, even if it doesn't appeal to you at all. I completely disagree with this opinion and think that job satisfaction is much more important than salary.

First of all, I believe that job satisfaction gives people a sense of fulfillment that no money can guarantee. Even if someone is earning a high salary, but feels tensed and compromises with his conscience, this person won’t enjoy his life. While pursuing one’s interests will always bring pleasure and feeling of satisfaction. For example, a lot of famous researchers made their career choices not because of appealing wages, but because they were passionate about science. That’s why it’s more important to choose the kind of work that makes you happy than to look only at a high salary.

Secondly, doing what you like keeps you motivated and therefore leads to a career growth. In other words, there is a strong relation between job satisfaction and productivity. People who love their jobs can easily excel in their fields of work and achieve better results than those, who put salary on the first place. For instance, Henry Miller decided to leave his everyday job despite a good wage and ventured to become a writer. And after enduring years of ups and downs he became one of the most famous and well-paid authors of the twentieth century. Thus, advantages of jobs that keep you satisfied outweigh the drawback of a low salary in a long-term perspective.

(277 words)

IELTS Agree or Disagree Essay

IELTS Agree or Disagree Essay

  • Opinion Essays
  • Discussion Essays
  • Problem Solution Essays
  • Advantages & Disadvantages Essays
  • Double Question Essays

Example of IETS Opinion essay

  • You should spend about 40 minutes on this task.
  • Write at least 250 words.
  • Give reasons for your answer and include any relevant examples from your knowledge or experience.

Structure of IELTS Opinion Essay

  • Paragraph 1: Introduction
  • Paraphrase the Essay Topic
  • Thesis Statement
  • Paragraph 2: Supporting Paragraph #1
  • Topic Sentence
  • Support (Example or Experience)
  • Explanation
  • Paragraph 3: Supporting Paragraph #2
  • Paragraph 4: Conclusion
  • Restate Thesis/Summarize your ideas

To what extent..

Example vocabulary and phrases: stating your opinion.

  • In my opinion I believe that education should be free because it helps move society forward.
  • As far as paying for education is concerned, I believe that you should have to pay because it creates competition which helps to develop stronger institutions.
  • My impression is that education should require a cost whether it be through tax payer money or private institutions.
  • Most institutions require tuition to attend. However. I believe that education should be free because it helps move society forward.

Example Vocabulary and Phrases: Arguing Your Point

  • This proves that free education can provide many opportunities for those who cannot afford it.
  • According to this, it can be argued that financial aid is a way to support students who cannot afford to pay for education.

Example Vocabulary and Phrases: Adding Adverb

Beginning of the sentence, middle of the sentence.

  • Clearly, this is an example of numerous afford a proper education.
  • This is definitely true because there are many students who are unable to attend school because they cannot afford it.
  • Deciding whether education should be free is absolutely a major challenge to consider.
  • The right to receive an education is substantially more important than earning money.

Example Vocabulary and Phrases: Verbs

  • I disagree that education should cost students.
  • I believe that education should be free.
  • I have no doubt that society will benefit with free education.
  • I think that education should be free.
  • It cannot be denied that education costs money because teachers, faculty, and staff all need to be paid.
  • As I see it, education has costs, so someone needs to pay for it.

Before You Start

  • Think about how you will plan to write your essay.
  • Brainstorm and generate ideas.

Outline structure for IELTS Essay

  • ________________________________

Outline structure for IELTS Opinion Essay

  • Paragraph I: Introduction
  • Paraphrase the Essay Topic - I believe that everyone should have access to free education without limitations.
  • Thesis Statement - Education is a valuable resource and it advances society.
  • Education is tool that helps us succeed
  • Germany - Free education
  • Same philosophy - society advances
  • Paying for education helps drive competition between institutions, but I believe this restricts social mobility.
  • Student cannot afford education
  • If every person of society is allowed to move forward, then all of society will benefit.
  • In conclusion education is essential to any society
  • By restricting access to it because of tuition limits societal advancements, it is important that we eliminate sort of barriers to education, Including costs.

Example Essay

  • Thesis Statement - Education is a valuable resource and it advances society. If there are certain barriers to receiving education like costs, many students would lose the opportunity to pursue an education because of this.
  • Education is a tool and it advances society and with free universal access to education, there are no limits to what a country and what a society can obtain.
  • A perfect example of this Is Germany, where universities are now tuition-free.
  • If all countries developed the same philosophy towards education as German. I have no doubt that society will benefit.
  • Some may argue that paying for education helps drive competition between institutions and helps to develop stronger schools. However. 1 completely disagree because I believe this restricts social mobility.
  • In other words, if a student who would like to pursue a degree In higher education, but cannot afford the high tuition rates then he or she will be unable to further their education.
  • On the other hand, If every person of society is allowed to move forward, then all of society will benefit.
  • In conclusion education is essential to any society.

Example Essay in color

agree or disagree essay outline

Share this article

If you liked this article share it with your friends. They will thank you later.

Write For Us

I am always looking for new guest authors and I welcome teachers and individual bloggers to contribute high-quality guest posts to writing9.com. Read more »

Ultimate IELTS Band 7+ Structure for Agree or Disagree Essay

The ultimate answer format that guarantees a 7+ Band in Agree or Disagree Essay

General Structure For Agree Or Disagree

When it comes to writing an agree or disagree essay in the IELTS exam, it is important to structure your essay effectively to present a clear and coherent argument. Here's a suggested structure to follow:

  • Paraphrase the question.
  • Begin with a captivating opening statement to engage the reader.
  • Clearly state your position on the given statement, indicating whether you agree or disagree.
  • Provide a brief overview of the main points you will discuss in the essay.
  • Start with a strong topic sentence that presents the main reason supporting your agreement or disagreement.
  • Provide supporting evidence, examples, or data to justify your position.
  • Elaborate on the implications or consequences of your viewpoint.
  • Address any potential counterarguments or opposing perspectives.
  • Begin with a clear topic sentence that introduces another reason supporting your agreement or disagreement.
  • Present supporting evidence, examples, or data to reinforce your stance.
  • Analyze the implications or consequences of this perspective.
  • Respond to any counterarguments or opposing viewpoints.
  • Summarize the main points discussed in the essay without introducing new information.
  • Restate your position clearly, emphasizing the strength of your argument.
  • Provide a concluding statement that reinforces your stance and highlights the broader significance or implications of the topic.

Remember to use appropriate vocabulary and linking words to express agreement or disagreement effectively. Examples of words and phrases for agreement include "indeed," "undoubtedly," "clearly," and "it is evident that." For disagreement, you can use words and phrases such as "however," "on the contrary," "it can be argued that," and "despite." Additionally, maintain a balanced and logical approach, supporting your position with well-reasoned arguments and evidence.

Fill In The Blanks

Here's a fill-in-the-blank structure for a Band 7+ Agree or Disagree essay to memorize as a backup.

Introduction: In today's complex society, the topic of ___________ has sparked intense debate. While some individuals espouse the view that ___________, others staunchly disagree, positing that ___________. This essay will meticulously examine both perspectives, evaluating their merits and demerits, in order to arrive at a well-founded position.

Body Paragraph 1: Proponents of the statement argue that ___________. They contend that ___________ because ___________. For instance, ___________. This exemplifies how ___________.

Body Paragraph 2: Conversely, critics maintain that ___________. They assert that ___________ due to ___________. Additionally, ___________. This highlights the viewpoint that ___________.

Body Paragraph 3: In my considered opinion, I (agree/disagree) with the statement. I firmly believe that ___________. Firstly, ___________. Moreover, ___________. Additionally, ___________. Thus, based on these cogent reasons, I (agree/disagree) with the statement.

Conclusion: In conclusion, the multifaceted debate surrounding ___________ embodies robust arguments from opposing perspectives. While proponents emphasize the advantages of ___________, critics raise pertinent concerns about ___________. After meticulous evaluation, I am inclined to (agree/disagree) with the statement. It is crucial to (support/oppose) ___________ in order to ___________.

Remember to fill in the blanks with relevant points and provide supporting evidence or examples for each argument you present. Incorporate advanced linking words and phrases to establish strong connections between your ideas. This structure, combined with comprehensive analysis and a well-reasoned position, can help achieve a Band 8-level essay.

Example Essay: Impact Of Social Media On Mental Health

Question: To what extent do you agree or disagree with the following statement: "Social media has a negative impact on mental health"?

In today's society, the topic of social media's impact on mental health has sparked significant discussion. While some argue that social media enhances connectivity and provides a platform for self-expression, others contend that it contributes to detrimental effects on mental well-being. This essay will delve into both perspectives, examining the potential benefits and drawbacks of social media on mental health.

Supporters of social media argue that it fosters connectivity and community. They contend that platforms such as Facebook, Instagram, and Twitter allow individuals to stay connected with friends and family, especially over long distances. Moreover, social media provides a space for marginalized communities to find support and representation. For instance, individuals struggling with mental health issues can connect with others who share similar experiences, offering a sense of belonging and understanding. This highlights the positive impact of social media on mental health.

Conversely, critics assert that social media contributes to various negative effects on mental well-being. They argue that excessive use of social media can lead to feelings of loneliness, inadequacy, and anxiety. The constant exposure to carefully curated and filtered versions of others' lives can create unrealistic expectations and trigger feelings of social comparison. Furthermore, cyberbullying and online harassment are prevalent issues that can significantly impact individuals' mental health. These concerns highlight the potential harm that social media can inflict.

In my perspective, it is essential to recognize both the benefits and drawbacks of social media on mental health. While social media has the potential to foster connection and support, it is crucial to use it mindfully and with awareness of its impact. Firstly, individuals should curate their online experiences by following accounts that promote positivity and well-being. Moreover, setting boundaries on social media usage, such as limiting screen time and taking breaks, can help maintain a healthy relationship with these platforms. Additionally, promoting digital literacy and educating users about responsible online behavior can mitigate the negative effects of social media.

In conclusion, the impact of social media on mental health is a complex issue with compelling arguments from both perspectives. While proponents emphasize the connectivity and support social media can offer, detractors raise valid concerns about its detrimental effects. It is crucial to approach social media use mindfully, curating positive online experiences, setting boundaries, and promoting digital literacy. By doing so, we can maximize the potential benefits of social media while safeguarding our mental well-being.

  • Ebooks & Courses
  • Practice Tests

How to Plan & Write IELTS Opinion Essays

IELTS opinion essays, also known as ‘agree or disagree’ essays, come up frequently in the writing exam. In this lesson, I’m going to show you how to plan and write them step-by-step.

Here’s what we’ll be covering:

  • 3 Common mistakes
  • Essay structure
  • How to plan
  • How to write an introduction
  • How to write main body paragraphs
  • How to write a conclusion

Click the links to see lessons on each of these Task 2 essay writing topics. 

Once you understand the process, practice on past questions. Take your time at first and gradually speed up until you can plan and write an essay of at least 250 words in the 40 minutes allowed in the exam.

The Question

The first part of the question for an IELTS opinion essay will be a statement. You will then be asked to give your own opinion about the statement. Here is some typical wording that might be used:

  • What is your opinion?
  • Do you agree or disagree?
  • To what extent do you agree or disagree?

Want  to watch and listen to this lesson?

Click on this video.

Here's a question from a past test paper.

A big salary is much more important than job satisfaction.  

Do you agree or disagree?  

Give reasons for your answer and include any relevant examples from your own knowledge or experience.

Write at least 250 words.

I’ll be using this question to guide you through the process of planning and writing an IELTS opinion essay.

3 Common Mistakes

These three errors are common in IELTS opinion essays.

  • Not stating an opinion.
  • Giving arguments for both views.
  • Not supporting your opinion with clear reasons.

The most common mistake that students make is not giving an opinion. The question will clearly state that you must choose one side of the argument. If you fail to do this, you will get a low score for task achievement.

It doesn’t matter which side of the argument you take or even, that you agree with it. Choose the one you can develop the best argument for.

Make sure that you don’t change your opinion part way through the essay, and don’t give reasons for the opposing view.

Essay Structure

Now let’s look at a simple structure you can use to write opinion essays. It’s not the only possible structure but it’s the one I recommend because it’s easy to learn and will enable you to quickly plan and write a high-level essay.

1)  Introduction

  • Paraphrase the question
  • Give your opinion
  • State two supporting reasons

2)  Main body paragraph 1

  • Topic sentence – outline 1st reason for supporting this view
  • Explanation – explain this idea
  • Example – give an example  or expand the idea

3)  Main body paragraph 2

  • Topic sentence – outline 2nd reason for supporting this view
  • Example – give an example or expand the idea

4)  Conclusion

  • Summarise opinion and key reasons

This structure will give us a well-balanced essay with 4 paragraphs.

We now need some ideas to add into the structure and we’ll have everything we need for our essay.

How To Plan IELTS Opinion Essays

# 1  decide on your opinion.

The question I've chosen to work on is quite straightforward and easy to understand so we don’t need to spend time analysing it. The first task, then, is to decide on our opinion.

Here’s the question again:

A big salary is much more important than job satisfaction.

Do you agree or disagree?  

For this essay, I’m going to disagree with the statement and argue that job satisfaction is more important than a big salary.

# 2  Generate ideas

The second task is to generate some ideas to write about.

Since I‘m going to argue that job satisfaction is more important than a large salary, I need ideas to support this view.

There are several different ways to think up ideas. I cover them fully on the  IELTS Essay Planning  page.

With this particular question, I immediately thought of a couple of examples of situations where job satisfaction did prove to be more important than a high salary, so I’m going to use the ‘example method’ of generating ideas.

Once you’ve thought of an example or two, ideas to include in your essay should come to you easily.

You might want to try this yourself before reading on for my ideas.

Here are my examples and some ideas they generated.

Both the examples are partly true but I've adapted them to better fit the essay. It's fine to do this as the examiner won't check your facts.

  • Uncle Barry – boasted about high salary but hated his job. Nervous breakdown – lost job & can’t work.
  • Me – gave up teaching. Now enjoy my work and am much more relaxed and happy even though I earn much less money.
  • High-salary jobs are generally more stressful
  • Stress leads to ill health, both mental and physical
  • 40 hours a week at work – a third of the day
  • Money doesn’t bring happiness
  • Better quality of life
  • Sense of fulfilment
  • Less stressed – healthier and happier

I’ve got more ideas here than I need so I’m going to pick two to develop in the essay – one for each of the main body paragraphs.

Idea 1 – High-salary jobs are generally more stressful and can lead to ill health.

Idea 2 – Job satisfaction gives a sense of fulfilment.

We’re almost ready to start writing our IELTS opinion essay but first, we have one other small task to do.

# 3  Vocabulary

In an IELTS essay, it’s important to be able to say the same things in different ways, either by paraphrasing and/or using synonyms. During the planning stage, quickly jot down a few synonyms of key words you could use to save you having to stop and think of the right language while you’re writing.

For example:

satisfaction – fulfilment, achievement, sense of accomplishment, content, sense of well-being

salary – income, wages, pay, earnings

important – significant, valued, has more meaning

job – work, employment, position

With that done, we can focus on the first paragraph of the essay – the introduction.

How To Write an Introduction

A good introduction has a simple 3 part structure:

1)  Paraphrased question

2)  Thesis statement

3)  outline statement.

An introduction should:

  • Have 2-3 sentences
  • Be 40-60 words long
  • Take 5 minutes to write

1)  Paraphrase the question

Start your introduction by paraphrasing the question.

     Question:  A big salary is much more important than job satisfaction.

                       Do you agree or disagree?  

Paraphrased question:  

It is argued that earning lots of money has more significance to people than being content in their work.

Note that I’ve used some of the synonyms I listed, although it’s fine to repeat one or two words if you need to. Above all, your language must sound natural.

In IELTS opinion essays, the thesis statement is where you state your opinion. For example,

    Thesis statement:  

    This essay totally disagrees with that statement.

That’s all you need to say.

If you decided to agree with the statement, you would write:

'This essay completely agrees with that statement.'

Finally in the introduction, you must outline the two main points (ideas 1 and 2 above) that you’ll cover in the rest of the essay. Do it in one sentence, or you can add them onto the end of the thesis statement if appropriate.

Outl ine statement:  

I believe that people are increasingly concerned about the risk of stress-related ill-health frequently experienced by people in highly paid positions and they care more about feeling fulfilled at work.

So, let’s bring the three elements of our introduction together.

     Introduction

agree or disagree essay outline

This introduction achieves three important functions:

  • It shows the examiner that you understand the question.
  • It acts as a guide to the examiner as to what your essay is about.
  • It also helps to keep you focused and on track as you write.

The two ideas in your introduction will become your two main body paragraphs.

Main body paragraph 1  – concerns about the risk of stress-related ill-health

Main body paragraph 2  – a sense of fulfilment at work

How To Write Main Body Paragraphs

The structure of a good main body paragraph has 3 parts:

  • Topic sentence
  • Explanation

If you can’t think of an example, you can add further supporting ideas but we already have our two examples so that’s not an issue here.

A common problem when writing main body paragraphs for IELTS opinion essays is having too many ideas. Again, we have already chosen the two ideas we are going to develop, so we are all set to start writing.

You can see how important the planning stage is and how it makes the actual writing of the essay far quicker and easier.

Main Body Paragraph 1

The  topic sentence  summarises the main idea of the paragraph. That’s all it needs to do so it doesn’t have to be complicated.

It plays an important role in ensuring that your ideas flow logically from one to another. It does this by acting as a signpost for what is to come next, that is, what the paragraph will be about.

If you maintain a clear development of ideas throughout your essay, you will get high marks for task achievement and cohesion and coherence.

We’ll now take the idea for our first main body paragraph and create our topic sentence.

Main idea 1  – concerns about the risk of stress-related ill-health

Topic sentence:  

Employees earning a large income are generally under significant mental and emotional pressure to perform well and achieve targets.

Next, we must write an  explanation sentence . This explains to the examiner what we mean. It expands on our first idea.

Explanation sentence: 

This causes many individuals to suffer high levels of stress which can result in both mental and physical health problems.

Finally, we add an  example  to support our main point. I thought of this in the planning stage so I have it ready to use.

If you can’t think of a real example, it’s fine to make one up, as long as it’s believable. The examiner isn’t going to check your facts.

Example sentence:

This happened to my uncle. He used to boast about his huge salary but the boss kept increasing his sales targets and in the end, the stress became too great and he had a nervous breakdown. Now he regrets being driven by the money.

That’s the 3 parts of our first main body paragraph complete. Here’s the finished paragraph.

agree or disagree essay outline

We now follow the same process for our second main body paragraph.

Main Body Paragraph 2

Main idea 2  – Job satisfaction gives a sense of fulfilment.

First, we write the  topic sentence  to summarise the main idea.

Topic sentence:

Having a job that they enjoy doing, and in which they feel valued, is a major concern for most of the modern workforce.

Now for the  explanation sentence  to explain this idea.

Explanation sentence:

A significant number of people are giving up well-paid positions to do jobs which pay less but that they find more enjoyable and less stressful.

Finally, an  example  to support our main point. As before, I thought of this in the planning stage so just need to form it into a couple of sentences.

I am an example of this myself. A year ago I left the teaching profession because the workload had become too great and I am now a gardener. I feel really fulfilled in this work and I am much more relaxed and happy even though I earn far less money.

That’s the 3 parts of our second main body paragraph complete. Here’s the finished paragraph.

agree or disagree essay outline

Now we need a conclusion and our IELTS opinion essay is done.

How To Write a Conclusion

Conclusions to IELTS opinion essays should do two things:

  • Summarise the main points
  • State your opinion

This can generally be done in a single sentence.

If you are below the minimum 250 words after you’ve written your conclusion, you can add an additional prediction or recommendation statement.

Our essay currently has 233 words so we’re on target and don’t need this extra sentence but you can learn more about how to write a prediction or recommendation statement for IELTS opinion essays on the Task 2 Conclusions page.

The conclusion is the easiest sentence in the essay to write but one of the most important.

A good conclusion will:

  • Neatly end the essay
  • Link all your ideas together
  • Sum up your argument or opinion
  • Answer the question

If you achieve this, you’ll improve your score for both task achievement and cohesion and coherence which together make up 50% of the overall marks. Without a conclusion, you’ll score below band 6 for task achievement.

You can start almost any final paragraph of an IELTS opinion essay with the words:

  • In conclusion

        or

  • To conclude

Now all you need to do is briefly summarise the main ideas into one sentence.

Here’s a top tip . Go back and read the introduction to the essay because this is also a summary of the essay. It outlines what you are going to write about.

To create a great conclusion, you simply have to paraphrase the introduction. Let’s give it a go.

Introduction:

agree or disagree essay outline

Here is the same information formed into a conclusion:

agree or disagree essay outline

That’s it. We’ve completed our essay. Here it is with the 4 paragraphs put together.

    Question:

   A big salary is much more important than job satisfaction.

   Do you agree or disagree?

Finished IELTS opinion essay.

agree or disagree essay outline

Go through this lesson as many times as you need to in order to fully understand it and put in lots of practice writing IELTS opinion essays from past exam questions. Practice is the only way to improve your skills.

5 More Model IELTS Opinion Essays

agree or disagree essay outline

This pack contains another step-by-step lesson and  model essay. P lus 4 additional opinion essay questions with model answers.

Carefully created to help you achieve 7+ in your Writing test.

Would you prefer to share this page with others by linking to it?

  • Click on the HTML link code below.
  • Copy and paste it, adding a note of your own, into your blog, a Web page, forums, a blog comment, your Facebook account, or anywhere that someone would find this page valuable.

Like this page?

More help with ielts opinion essays & other task 2 essays.

IELTS Writing Task 2  – T he format, the 5 question types, the 5 step essay writing strategy & sample questions. All the key information you need to know.

The 5 Types of Task 2 Essay   – How to recognise the 5 different types of Task 2 essays. 15 sample questions to study and a simple planning structure for each essay type.

Understanding Task 2 Questions  – How to quickly and easily analyse and understand IELTS Writing Task 2 questions.

How To Plan a Task 2 Essay  – Discover why essay planning is essential & learn a simple 4 step strategy, the 4 part essay structure & 4 methods of generating ideas.

How To Write a Task 2 Introduction  – Find out why a good introduction is essential. Learn how to write one using a simple 3 part strategy & discover 4 common mistakes to avoid.

How To Write Task 2 Main Body Paragraphs  – Learn the simple 3 part structure for writing great main body paragraphs and also, 3 common mistakes to avoid. 

How To Write Task 2 Conclusions  – Learn the easy way to write the perfect conclusion for a Task 2 essay. Also discover 4 common mistakes to avoid.

Task 2 Marking Criteria  – Find out how to meet the marking criteria in Task 2. See examples of good and poor answers & learn some common mistakes to avoid.

The 5 Task 2 Essay Types:

Step-by-step instructions on how to plan & write high-level essays. Model answers & common mistakes to avoid.

   Opinion Essays

   Discussion Essays

  Problem Solution Essays

  Advantages & Disadvantages Essays

  Double Question Essays

Other Related Pages

IELTS Writing Test  – Understand the format & marking criteria, know what skills are assessed & learn the difference between the Academic & General writing tests.

  • IELTS Writing
  • Opinion Essays
  • Back To Top

 * New * Grammar For IELTS Ebooks

agree or disagree essay outline

$9.99 each       Full Set   Just   $ 23.97

Find Out More >>

IELTS Courses

agree or disagree essay outline

Full details...

agree or disagree essay outline

IELTS Writing Ebook

agree or disagree essay outline

Discount Offer

$7 each       Full Set Just   $ 21

agree or disagree essay outline

Find out more >>

Testimonials

“I am very excited to have found such fabulous and detailed content. I commend your good work.”  Jose M.

“Thanks for the amazing videos. These are ‘to the point’, short videos, beautifully explained with practical examples."  Adari J.

"Hi Jacky, I bought a listening book from you this morning. You know what? I’m 100% satisfied. It’s super helpful. If I’d had the chance to read this book 7 years ago, my job would be very different now."  Loi H.

"Hi Jacky, I recently got my IELTS results and I was pleased to discover that I got an 8.5 score. I'm firmly convinced your website and your videos played a strategic role in my preparation. I was able to improve my writing skills thanks to the effective method you provide. I also only relied on your tips regarding the reading section and I was able to get a 9! Thank you very much." Giano

“After listening to your videos, I knew I had to ditch every other IELTS tutor I'd been listening to. Your explanations are clear and easy to understand. Anyways, I took the test a few weeks ago and my result came back: Speaking 7, listening 9, Reading 8.5 and Writing 7 with an average band score of 8. Thanks, IELTS Jacky." Laide Z.

      Contact

      About Me

      Site Map

      Privacy Policy

      Disclaimer

IELTS changes lives.

Let's work together so it changes yours too.

Copyright  © 2024     IELT Jacky     

All Right Reserved

IELTS is a registered trademark of the University of Cambridge, the British Council, and IDP Education Australia. This site and its owners are not affiliated, approved or endorsed by the University of Cambridge ESOL, the British Council, and IDP Education Australia.

Create profile

Like courses

Apply direct

Create your IDP profile

To continue your research, create your profile with IDP. Your profile allows you to:

  • Apply direct to courses and receive a response within the same day
  • Shortlist and save courses
  • Get the AI course recomendations
  • Access our cost of living calculator

Match with universities

Now create a profile

Create a profile and start liking courses. We’ll show you recommendations that match what you’re looking for.

Your password must include

  • One upper case letter
  • One lower case letter
  • One special character
  • At least 8 characters

Business Hours

Monday-Friday 1PM-9PM EST

Saturday-Sunday 11AM-6PM EST 

A Simple Formula for Organizing Agree/Disagree Essays

agree or disagree essay outline

When you get to the essay writing component of the IELTS exam, the clock is ticking and the pressure is on. You have about 40 minutes to determine the key question to address, think of your response, come up with relevant examples and then write the essay.

Wouldn’t it be nice if you knew how you were going to organize your ideas before you went into the exam? In this blog, I’m going to show you a formula that can be modified to fit most IELTS tasks and will save you valuable time. Most importantly it will ensure that the message in your essay is easy to follow (one of the key criteria for higher level scores).

agree or disagree essay outline

Today it is a popular belief that students attending high school should volunteer in the community as part of their schooling. I strongly agree with this notion. Volunteer work can provide young people with the experiences they need for paying jobs and it can also help them see themselves as valuable contributors to society.

When completing compulsory work activities with charitable organizations or community organizations, students gain skills that they can add to their resumes and use in future jobs. Working out in the real world quickly teaches young people the importance of time management, clear communication and teamwork. Employers like McDonald’s restaurants, who provide many young people with their first paying jobs, look for volunteer work on resumes as evidence of having developed some relevant work skills. So, although students are not paid for their time, they are building valuable abilities for future employment.

Perhaps even more importantly, when students volunteer, they are more likely to realize that their actions have a positive impact on the world around them. Often the problems of our world can seem overwhelming but if students are given the opportunity to help others, even in a small way, they will see that they can make a difference. In schools that have been running these programs, there has been an incredible boost in student morale and the community has benefitted from the talent and time that young people have contributed to the many programs that need support. It’s true that studying for academics is time-consuming and students already have a lot to do in a day but if we teach them how to step up and help others, we create a better future for us all.

In conclusion, it is a great idea to have unpaid community service as part of a high school program because it will help the next generation prepare for employment and it will build a caring community that is willing and able to make our world a better place. It’s my hope to see this in every school.

Now, let’s look at the underlying structure sentence by sentence.

AGREE/DISAGREE ESSAY - 4 PARAGRAPH FORMULA Paragraph One - Introduction

  Paragraph Two - Body

Paragraph Three - Body  – Repeat the above

Paragraph Four – Conclusion

What I hope you see from this example is that when you pay attention to how you organize your essay, it’s easier to read. I also want you to realize that it isn’t difficult to accomplish this clarity when you have a reliable structure in mind.

Check back for future blogs that will show you how to modify this formula for other kinds of IELTS essay tasks.

In the meantime, here is a blank template for you to use when you write your next agree/disagree essay.

AGREE/DISAGREE 4 PARAGRAPH FORMULA

Paragraph One -Introduction

Sentence 1:

___________________________________________________________________________

State the premise that the essay will address (you can simply rephrase the words in the task).

Sentence 2:

State your opinion in the matter.

Sentence 3:

Briefly outline what you are going to write in the next two paragraphs to support your opinion.

Paragraph Two and Three – the Body of Support

Sentence 1 (topic sentence):

Tell the reader what this paragraph is about by introducing your first point .

Assume that the reader doesn’t know anything about this topic and explain your point clearly.

Give a real life or made up example that illustrates and supports this point.

Sentence 4:

Concede to a relevant opposing position but redirect to yours.

Paragraph Three – Repeat the above

Sentence 1 (topic sentence)

Tell the reader what this paragraph is about by introducing second point .

Give a real life or made up example that illustrates and supports this point .

Rephrase your opinion and sum up your two supporting points.

Make a prediction or a recommendation based on what you have said.

By Angela Rutherford

Recent posts

  • March 2024 1
  • December 2023 4
  • October 2023 4
  • September 2023 11
  • August 2023 5
  • June 2023 1
  • March 2023 3
  • February 2023 2
  • January 2023 1
  • View All 181

Recommended articles

Find Your Course

With so many reputable schools and courses in Canada, deciding where and what to study can be an overwhelming decision. Chat today with an IDP expert who can guide you through your study options and assist with your application.

Our History

We are here to help connect people like you to schools across Canada. Our experience in Canadian education translates into the best outcome for you.

  • Search for courses
  • Find a university
  • Find a scholarship

Please select a level of study

Enter subject, choose from the list or hit search

Start typing, choose from the list or hit search

Enter subject, choose from the list or or hit search

Please type and select an institution

Type 1 character of a university name and select from the list

Enter a university or school name and select from the list

Got any ideal countries in mind?

No Event Found.

Let’s get started

Sign up or login in with one click, sign up or login to save your courses, let's get started with "shortlist".

Your profile page will have the liked courses.

has been saved to your shortlist

View your shortlist or close this box to continue researching.

  • Courses for you

agree or disagree essay outline

25,000+ students realised their study abroad dream with us. Take the first step today

Meet top uk universities from the comfort of your home, here’s your new year gift, one app for all your, study abroad needs, start your journey, track your progress, grow with the community and so much more.

agree or disagree essay outline

Verification Code

An OTP has been sent to your registered mobile no. Please verify

agree or disagree essay outline

Thanks for your comment !

Our team will review it before it's shown to our readers.

Leverage Edu

  • Study Abroad Test Prep /

IELTS Agree/Disagree Essay: How to Write, Structure, Tips, Sample Answers 

' src=

  • Updated on  
  • Jan 2, 2024

agree and disagree essay ielts

IELTS General Writing Task 2 presents a challenge for test takers to craft comprehensive essays. One of the most recurring topics in this segment is the agree/disagree essays. Such essays prompt you to articulate your stance on a given topic, demanding concise, clear, and precise expression. Essentially, you aim to present your reasoned opinions with clarity and avoid redundancy or errors in grammar and spelling. For those seeking to delve deeper into the art of crafting impactful agree/disagree essays for IELTS writing task 2, this comprehensive article offers invaluable insights and guidance. Read the article to know more. 

This Blog Includes:

Ielts agree/disagree essay structure , how to write an agree/disagree essay for ielts writing task 2 , ielts agree/disagree essay: do’s and don’ts, ielts agree/disagree essay: sample questions with answers .

Mastery of structure is paramount for acing the IELTS agree/disagree essays. A thorough understanding of this format facilitates clear organisation and articulation of your opinions and arguments, leading to a more comprehensive and coherent response. That being said, have a look below to go through the IELTS agree/disagree essay structure. 

IELTS writing task 2 frequently features agree/disagree essays. Mastering this format is crucial for success, as these essays offer significant scoring potential. A strong understanding of structure and effective argumentation can pave the way for high band scores. In IELTS Writing Task 2, some essay prompts present a statement and require you to express your level of agreement or disagreement. With that in mind, let’s explore the key elements of crafting a successful agree/disagree essay for this task.

Introduction

This is the most crucial segment of any essay. You must ensure that your introduction is short, crisp and conveys your message. Two sentences are enough here. Whilst writing an agree/disagree essay, always start your introduction by paraphrasing the task question. Once you have paraphrased the task question, you must state your opinion on the topic, i.e., whether you agree or disagree with the given statement. This will let your reader know what you think immediately. 

Body Paragraphs 

Strong paragraphs play a critical role in structuring, articulating, and ensuring the coherence and comprehension of your agree/disagree essays in IELTS Writing Task 2. Fundamentally, three approaches exist:

  • Full Agreement: In the first paragraph, present your compelling arguments supporting the statement. The second paragraph can further reinforce your position with additional justifications.
  • Full Disagreement: Dedicate the first paragraph to clearly outline your counter-arguments against the statement. The second paragraph can strengthen your stance by introducing further dissenting points.
  • Partial Agreement : The first paragraph should encompass reasons for aligning with certain aspects of the statement. The second paragraph then transitions to explore reasons for disagreeing with other aspects, ultimately presenting a nuanced perspective.

Your conclusion should be crisp and to the point. Brevity can be your ally in this case. One golden rule for writing impactful conclusions is to keep them one to two sentences long. Your conclusion should summarise the entirety of your essay in just a few sentences. Apart from that, you should avoid introducing new information in conclusions. Therefore, it is ideal to not introduce other reasons or topics at the end of your text.

Here is a list of do’s and don’ts that you must keep in mind while writing an IELTS agree/disagree essay. 

Here are the best practices for writing an impactful agree/disagree essay in IELTS writing task 2. 

  • In your introduction, leave no ambiguity concerning your level of agreement or disagreement with the prompt.
  • Structure your arguments logically. Organise your supporting points into distinct paragraphs, ensuring a coherent flow of ideas.
  • Back up your arguments with relevant examples, statistics, or research findings to bolster your credibility.
  • Engage in nuanced analysis. Beyond simply stating your stance, explore potential counter-arguments or exceptions to strengthen your overall position.
  • Employ a range of sentence structures to maintain reader engagement and showcase your language skills .
  • Maintain strong grammar and vocabulary. Demonstrate proficiency in English language mechanics to avoid detracting from your content.
  • Limit yourself to writing only four paragraphs.
  • Write a short conclusion.

  Here are the things you must avoid whilst writing an agree/disagree essay in IELTS writing. 

  • Avoid ambiguity in your introduction. Ensure to make them as concise as possible.  
  • Avoid jumping between points or omitting transitions. Doing so can confuse the reader and weaken your argument.
  • Assertions without supporting evidence appear unsupported and undermine your credibility.
  • Monotonous writing can bore the reader and limit your ability to showcase language proficiency.
  • Grammatical errors or vocabulary mistakes can significantly lower your band score, regardless of the strength of your arguments.
  • Avoid including too many different ideas. 

Sample IELTS Agree/Disagree Essays: Here are some sample IELTS agree/disagree essays that you can refer to whilst attempting the IELTS Writing Task 2. 

Question : When new towns are planned, it is important to build more public parks or sports facilities than shopping centres for people to spend their free time. To what extent do you agree or disagree?

Ans: The discussion that governments must spend more money on railways rather than roads is a complex one. While railways do have certain advantages, I feel that roads should be given more priority because of several reasons. 

Roads serve as the backbone of any nation’s transportation system by helping them interconnect cities, towns, and remote areas to ensure accessibility. On the other hand, railways have limited reach and mostly cater to urban and semi-urban regions. Take India as an example where a significant amount of the population resides in rural areas where roads are vital, for commuting, transporting goods, and accessing essential services like healthcare and education. In addition to this, roads accommodate a range of vehicles ranging from bicycles and cars to buses and trucks. This versatility is not replicated by railways that primarily focus on long-distance travel and bulk transportation of goods. Therefore, roads play a crucial role in both city and inter-city transportation. 

Finally, the construction and upkeep of roads typically demand lower costs and shorter timeframes in comparison to the development of railways. This inherent efficiency in road infrastructure allows governments to establish and maintain a more expansive network with the same financial resources. The flexibility and adaptability of road systems contribute to their cost-effectiveness, enabling quicker implementation of projects and more immediate responses to changing transportation needs. This advantage becomes particularly crucial in regions where swift infrastructure development is essential for economic growth and accessibility.

In conclusion, while railways hold importance in a country’s infrastructure and overall development, the advantages and adaptability of roads make them more deserving recipients of government funding. Hence, I respectfully disagree with the notion that governments should prioritise spending on railways over roads.

Question: Governments should spend money on railways rather than roads.

To what extent do you agree or disagree with this statement?

Ans : The discussion that governments must spend more money on railways rather than roads is a complex one. While railways do have certain advantages, I feel that roads should be given more priority because of several reasons. 

So that was all about IELTS Agree/Disagree Essays. Hope the blog has answered your queries regarding the topic. 

Ans. You can write a formal letter by including an introduction (Dear Sir/Madam), body paragraphs (1, 2,3 and more), and a signoff (I hope to hear from you, Yours faithfully). 

Ans. Test takers can use a range of linkers, adverbial phrases, references, and punctuations to bolster their overall scores in IELTS writing. 

Ans. The IELTS General Writing Task 1 (Letter Writing) requires individuals to complete the task within 20 minutes. 

Related Reads:

Visit the Leverage Live page of Leverage edu or contact our study abroad experts at 1800-57-2000 to strengthen your scores and application to secure your spot in your dream college. 

' src=

Shubham Das

Shubham Das has been working as an educational content writer for the past two years and has a background in filmmaking & screenplay/ teleplay writing. He is fascinated by the human psyche, literature and cinema.

Leave a Reply Cancel reply

Save my name, email, and website in this browser for the next time I comment.

Contact no. *

agree or disagree essay outline

Connect With Us

25,000+ students realised their study abroad dream with us. take the first step today..

agree or disagree essay outline

Resend OTP in

agree or disagree essay outline

Need help with?

Study abroad.

UK, Canada, US & More

IELTS, GRE, GMAT & More

Scholarship, Loans & Forex

Country Preference

New Zealand

Which English test are you planning to take?

Which academic test are you planning to take.

Not Sure yet

When are you planning to take the exam?

Already booked my exam slot

Within 2 Months

Want to learn about the test

Which Degree do you wish to pursue?

When do you want to start studying abroad.

January 2024

September 2024

What is your budget to study abroad?

agree or disagree essay outline

How would you describe this article ?

Please rate this article

We would like to hear more.

Have something on your mind?

agree or disagree essay outline

Make your study abroad dream a reality in January 2022 with

agree or disagree essay outline

India's Biggest Virtual University Fair

agree or disagree essay outline

Essex Direct Admission Day

Why attend .

agree or disagree essay outline

Don't Miss Out

Welcome Guest!

  • IELTS Listening
  • IELTS Reading
  • IELTS Writing
  • IELTS Writing Task 1
  • IELTS Writing Task 2
  • IELTS Speaking
  • IELTS Speaking Part 1
  • IELTS Speaking Part 2
  • IELTS Speaking Part 3
  • IELTS Practice Tests
  • IELTS Listening Practice Tests
  • IELTS Reading Practice Tests
  • IELTS Writing Practice Tests
  • IELTS Speaking Practice Tests
  • All Courses
  • IELTS Online Classes
  • OET Online Classes
  • PTE Online Classes
  • CELPIP Online Classes
  • Free Live Classes
  • Australia PR
  • Germany Job Seeker Visa
  • Austria Job Seeker Visa
  • Sweden Job Seeker Visa
  • Study Abroad
  • Student Testimonials
  • Our Trainers
  • IELTS Webinar
  • Immigration Webinar

ielts-material

Agree Disagree Essays in IELTS

Janet

Updated On Oct 13, 2023

agree or disagree essay outline

Share on Whatsapp

Share on Email

Share on Linkedin

Agree Disagree Essays in IELTS

Limited-Time Offer : Access a FREE 10-Day IELTS Study Plan!

IELTS Writing Task 2 Agree or Disagree Essay

The agree or disagree essay questions are the most commonly asked in the IELTS examination . They are also called as argumentative essays.  In this type of essays, you are asked to give your opinion, whether you agree or disagree on the particular sentence that is given. You are given 40 minutes to complete the 250-word essay. Most of the students find it easy because you only have to agree and disagree with the sentence given. You have to support only one opinion.

Tips to write Agree Disagree Essays

There are few tips which will  help you to write about IELTS agree disagree essay questions, which are as follows:

  • When you get these type of questions, it is advisable to completely agree or completely disagree with it. You have to support only one point of view and not both.
  • By doing this, the essay will be more clear and structured and can be easily understood by the examiner.
  • You can support the other side of the statement if you want to, but make sure that you do it in a positive way and also say it, only in one or two sentences.
  • While writing the essay, make sure that the explanation given doesn’t hurt the feelings of the other person.
  • The choice of agreement or disagreement has to be made carefully. You need to choose the side which has more supporting points and which you can illustrate with examples so that the answer that you write is voluminous. Please note that you have to either agree or disagree and cannot do both.
  • Also, ensure that the introduction of the topic is not too long. The question should be paraphrased to form the introduction.
  • The explanation that you give regarding the topic should match with the introduction and the conclusion given.
  • There will be two or three body paragraphs, make sure that each paragraph has a central topic.
  • The body paragraphs should include the main features that have to be explained and also your overview and an in-depth explanation of your opinion.
  • You have to compulsorily write the conclusion. If you do not have time to complete the essay, make sure you at least write 2 or 3 sentences of conclusion, but the conclusion should be included.

IELTS Agree Disagree Essay Structure

In all the sections of the IELTS examination, structure plays a very important role because it will be easy for the test taker to write the answer. Also, the examiner will find it easy to evaluate the answer. So here is the structure that you can follow while writing an agree/disagree essay:

Paragraph 1: Introduction

How to start an agree disagree essay?

First of all, like any other section, the first sentence must be a paraphrase of the question that is given. In the second sentence, you can state your opinion regarding the topic. In the third sentence, you can write on what you are going to explain in the forthcoming paragraphs.

Paragraph 2: Body paragraph

In the second paragraph, the first sentence should be explaining the central idea. In the second sentence, explain in detail, assuming that the examiner doesn’t know anything about the topic. In the third sentence, you can give examples which support the sentence. In the last sentence, you can give the opposing point of view, but in a positive manner.

Paragraph 3: Body paragraph

The third paragraph is very similar to the second paragraph. The first sentence should explain the central idea. The second sentence, should include  supporting details and the third sentence, examples which support the idea.

Paragraph 4: Conclusion

In the last paragraph, you have to write a summary of the essay. In the first sentence, you can sum up the body paragraphs and rephrase your opinion.

Sample Checklist for Agree/Disagree essays:

Agree disagree essay topics:.

  • Some people who have been in prison become good citizens later. Some people think that inviting these people to talk to school students is the best way to tell them about dangers of committing a crime. Do you agree or disagree?
  • The only way to improve road safety is to give much stricter punishments on driving offenses. To what extent do you agree or disagree?
  • It is generally accepted that exercise is good for children and teenagers. Therefore, physical education and sport should be compulsory for all students in all schools. What do you think?

Take a look at some IELTS Agree/Disagree Essay sample answer

Also check :

  • IELTS Writing tips
  • IELTS Writing recent actual test
  • IELTS Writing Answer sheet
  • IELTS map vocabulary

Frequently Asked Questions

Can agree/disagree essays be called as opinion essays?

Can I partially agree?

Can I skip the conclusion part if I’m running out of time. What weightage does that one liner consist of?

Can I use the word “I” such as ” I agree” or “I disagree”?

What is the structure of Agree/Disagree essays?

Practice IELTS Writing Task 2 based on Essay types

ielts img

Start Preparing for IELTS: Get Your 10-Day Study Plan Today!

Janet

Janet had been an IELTS Trainer before she dived into the field of Content Writing. During her days of being a Trainer, Janet had written essays and sample answers which got her students an 8+ band in the IELTS Test. Her contributions to our articles have been engaging and simple to help the students understand and grasp the information with ease. Janet, born and brought up in California, had no idea about the IELTS until she moved to study in Canada. Her peers leaned to her for help as her first language was English.

Explore other Writing Articles

Crime Novels and TV Crime Dramas are Becoming Popular – IELTS Writing Task 2

Kasturika Samanta

21+ Tips On How to Improve Your IELTS Writing Band Score

Raajdeep Saha

IELTS Essays on Technology – Discussion and Opinion Essays

Post your Comments

Recent articles.

Some People Prefer to Eat at Restaurants While Others Prefer to Prepare and Eat at Home – IELTS Writing Task 2

Nehasri Ravishenbagam

Many People Complain that They Have Difficulties Getting Enough Sleep – IELTS Writing Task 2

Our Offices

Gurgaon city scape, gurgaon bptp.

Step 1 of 3

Great going .

Get a free session from trainer

Have you taken test before?

Please select any option

Get free eBook to excel in test

Please enter Email ID

Get support from an Band 9 trainer

Please enter phone number

Already Registered?

Select a date

Please select a date

Select a time (IST Time Zone)

Please select a time

Mark Your Calendar: Free Session with Expert on

Which exam are you preparing?

Great Going!

TED IELTS

  • A Beginner’s Guide to IELTS
  • Common Grammar Mistakes [for IELTS Writing Candidates]

Writing Correction Service

  • Free IELTS Resources
  • Practice Speaking Test

Select Page

How to Start an Agreement Essay

Posted by David S. Wills | Aug 7, 2023 | IELTS Tips , Writing | 0

How to Start an Agreement Essay

In IELTS writing task 2, you could be asked to write an agree or disagree essay, also known as an “agreement essay.” Today, however, I want to show you how to start an agreement essay . This will focus on the first few lines.

You can read my full guide to agree/disagree essays here if you want to learn about the overall structure.

What are the requirements of an agreement essay?

First of all, let’s look at what an agreement essay actually is and what you need to do. Here’s an example:

All cars that burn fossil fuels should be banned and electric cars should replace them. Do you agree or disagree?

An IELTS agreement essay will typically give you a statement and then a line that says something like:

Do you agree or disagree?

To what extent do you agree or disagree with this opinion?

  • To what extent do you agree or disagree with the statement?

the structure of an ielts agree/disagree question

Your objective is to state whether (or to what extent) you agree or disagree with the idea expressed in the first line of the question. For the above example, you would either:

  • Completely agree
  • Partly agree
  • Neither agree nor disagree
  • Partly disagree
  • Completely disagree

Your essay should explain your position and your view should be clear throughout the entire piece of writing.

Now let’s look at the first lines.

How to start an agreement essay

Considering the above, what should you write in the opening lines of your essay?

Generally, an IELTS task 2 introduction should feature 2-4 sentences that clearly tell the reader what the essay is about and what your opinion is. You should write 1-3 sentences that explain the overall idea (basically introducing the topic) and then you should write an outline sentence .

Important: Your opinion must be stated in the introduction . Regardless of whether you agree, disagree, or have no strong opinion either way, you must make this clear to the reader. If you fail to do this, you will not score band 7 or above.

Thus, here is how you should start an agreement essay:

You often hear people say that you should paraphrase the question. This is certainly a possible approach, but it presents many problems and often results in terrible first sentences. In fact, it can ruin your whole introduction. You can learn about paraphrasing here .

Examples of agreement essay introductions

To further explain how you can start an agreement essay, let’s look at some examples.

All cars that burn fossil fuels should be banned and electric cars should replace them.

Intro to sample band 9 answer:

During the twentieth century, people around the world began using vehicles powered by fossil fuels, and in this new millennium that trend has continued, with a vast uptick in the number of privately owned cars. However, it has become apparent that this phenomenon is causing major environmental damage and needs to be stopped. This essay will argue that humans ought to switch to electric cars.

Instead of paraphrasing the question, I have taken the approach of explaining the wider situation. This is much better! I have used the next sentence to give a more focused introduction, raising the idea of banning traditional cars. Then, I gave my opinion. It is nice and clear and the reader would understand the topic fully before reading the body paragraphs.

A person’s worth nowadays seems to be judged according to social status and material possessions. Old-fashioned values, such as honour, kindness and trust, no longer seem important.

There are numerous ways in which a person can be valued, and these vary widely from culture to culture. They also vary over time, and some people suggest that in the modern era it is more common for people to be judged according to their wealth, rather than personality attributes. This essay will argue that it is probably not true.

Again, you can see that I have avoided blindly paraphrasing the question. Instead, I started with a nice, broad overview of the main idea, then focused it specifically in the next sentence. My opinion is made very clear in the final sentence.

Note: This is a notoriously difficult question and many people struggle with it. I’ve made this video to explain why it is so hard and how people can answer it intelligently.

Using a computer every day can have more negative than positive effects on young children.

Nowadays, it is quite common for children to have access to some kind of computer, and some people argue that this is damaging for their health. This essay will look at both sides of the argument, but ultimately conclude that it is not entirely negative.

This introduction is a little more “standard” in that I have sort of paraphrased the question. Still, it is hard to see that it is paraphrased because it is totally my words and ideas. I have then given my opinion clearly. Note that this may seem like an introduction to a “ discuss both views ” question but really I just wanted to talk about both sides because I feel that there is no clear-cut answer here. In other words, I’m providing balance because I neither wholly agree nor disagree.  

Question #4

Some people think that women should not be allowed to work in the police force.

Traditionally, there were some jobs that could only be done by men. These days, however, it is common to see women doing jobs that they were once forbidden from doing. Some people still hold onto the traditional view in regards the police force, but this essay will argue that women shouldn’t be unfairly restricted.

This goes back to my favoured approach, which is starting with a general statement, then refining it to a more specific point. As always, I have made my opinion very clear in the essay outline sentence. Note that I have avoided personal pronouns by saying “this essay will…” This is a good way to increase the level of formality without making it convoluted.

When you start your agreement essay, you should introduce the main topic and then provide your opinion. That is the key to writing a great introduction. Keep it relatively simple and make sure that your primary aim is to let the reader know exactly what your position is. Don’t try to show off with any fancy language or complicated ideas. Just make it straightforward and your essay will be off to a great start!

About The Author

David S. Wills

David S. Wills

David S. Wills is the author of Scientologist! William S. Burroughs and the 'Weird Cult' and the founder/editor of Beatdom literary journal. He lives and works in rural Cambodia and loves to travel. He has worked as an IELTS tutor since 2010, has completed both TEFL and CELTA courses, and has a certificate from Cambridge for Teaching Writing. David has worked in many different countries, and for several years designed a writing course for the University of Worcester. In 2018, he wrote the popular IELTS handbook, Grammar for IELTS Writing and he has since written two other books about IELTS. His other IELTS website is called IELTS Teaching.

Related Posts

Don’t Say Delicious! How to Really Talk About Food

Don’t Say Delicious! How to Really Talk About Food

March 23, 2017

IELTS Topics: Mobile Phones [Speaking and Reading Lesson]

IELTS Topics: Mobile Phones [Speaking and Reading Lesson]

June 3, 2020

Talking About Movies for IELTS

Talking About Movies for IELTS

November 28, 2017

Describe an App [IELTS Speaking]

Describe an App [IELTS Speaking]

November 2, 2020

Leave a reply Cancel reply

Your email address will not be published. Required fields are marked *

This site uses Akismet to reduce spam. Learn how your comment data is processed .

Download my IELTS Books

books about ielts writing

Recent Posts

  • How to Improve your IELTS Writing Score
  • Past Simple vs Past Perfect
  • Complex Sentences
  • How to Score Band 9 [Video Lesson]
  • Taxing Fast Food: Model IELTS Essay

ielts writing correction service

Recent Comments

  • abdelhadi skini on Subordinating Conjunction vs Conjunctive Adverb
  • David S. Wills on How to Describe Tables for IELTS Writing Task 1
  • anonymous on How to Describe Tables for IELTS Writing Task 1
  • David S. Wills on Writing Correction Service
  • James Oluwasegun on Writing Correction Service
  • Lesson Plans
  • Model Essays
  • TED Video Lessons
  • Weekly Roundup
  • Skip to primary navigation
  • Skip to main content
  • Skip to primary sidebar
  • Skip to footer

IELTS Advantage

IELTS Advantage

IELTS Preparation Courses

IELTS Writing Task 2 Essay Structures

The four most common IELTS writing Task 2 questions are: Opinion, Advantages and Disadvantages, Problem and Solution Discussion

IELTS Task 2 Essay Structures

Knowing how to structure your IELTS Writing Task 2 essay is an essential skill that can make the difference between getting and not getting the band score you deserve. With that in mind, we have outlined the most common IELTS Writing Task 2 structures below.

agree or disagree essay outline

Nearly all of my Task 2 essays follow this basic structure: The sentences you put in each paragraph will depend on what type of question you get.

The five most common IELTS Writing Task 2 questions are:

  • Opinion (Agree or Disagree)
  • Advantages and Disadvantages
  • Problem and Solution
  • Discussion (Discuss both views)
  • Two-part Question

Below I will outline examples and a structure approved by experienced IELTS teachers and examiners for each type of question. This will help you write a clear, coherent answer and hopefully boost your IELTS band score. I also include an example answer for each type of question so you can see the structure in a real essay.

Please note that these are general structures and may vary slightly depending on the question.

Please also note that no ‘one’ Task 2 essay structure will get you a high score. There are many types of structures that can get you a high score. These are just some I think are effective and easy to learn. 

Please visit the lessons below for more detailed guidance on each type of question. I have provided a link at the end of each section.

agree or disagree essay outline

Opinion Questions (Agree or Disagree) 

Typical Question Words –

What is your opinion?

Do you agree or disagree?

To what extent do you agree or disagree?

Direct question.

Example Question –

Some people believe that unpaid community service should be compulsory in high school programmes (for example, working for a charity, improving the neighbourhood or teaching sports to younger children).

Essay Structure 

Introduction 

1- Paraphrase Question

2- Give your opinion and outline the main ideas.

Main Body Paragraph 1 

1- Topic Sentence

2- Explain Topic Sentence

Main Body Paragraph 2

Conclusion 

1- Summary of main points and opinion

Student Sample Answer

It is argued that volunteering should be made part of the school curriculum. This essay agrees with that suggestion completely because it help pupils develop soft skills and helps them gain much-needed work experience.

Education should not be limited to strictly academic pursuits, and those in education should also develop life skills, such as teamwork, empathy and self-discipline, and one of the best ways to hone these aptitudes is through community service. Serving those less fortunate than ourselves teaches us many lessons, including how to work with people from other backgrounds and the value of hard work, thus enabling us to hone these skills before becoming an adult. For example, many young people from wealthier countries take a gap year and help those less fortunate than themselves to increase their gratitude for what they have and improve their work ethic.

Many colleges and companies are also increasingly looking for this type of experience. Most school leavers have the same grades, and charitable work can help set you apart from other students when making college applications. For example, Cambridge and Oxford receive thousands of applications from straight-A students yearly and can only accept a small percentage of applicants. What you have done outside the classroom often differentiates you from everyone else and gets you that coveted spot.

In conclusion, teenagers should be made to partake in unpaid work as part of their schooling because it will help them learn things they wouldn’t ordinarily learn from their teachers, and it will also boost their chances of getting into third-level education.

For more detail on how to answer agree or disagree questions, please visit our opinion essay lesson . 

Need help writing essays like this? Check out our ESSAY CORRECTION SERVICE .

Advantages and Disadvantages Questions

Typical Question Words 

Discuss the advantages and disadvantages.

What are the advantages and disadvantages?

Example Question

Technology is being used more and more in education.

Essay Structure

2- Outline Main Points

Main Body Paragraph 1

1- State Two Advantages

2- Expand/Explain First Advantage

3- Expand/Explain Second Advantage

1- State Two Disadvantages

2- Expand/Explain First Disadvantage

3- Expand/Explain Second Disadvantage

agree or disagree essay outline

1- Summary of Main Points

Student Sample Answer 

It is argued that technology plays an ever-increasing role in schools and universities. Increased access to information and student freedom are the main advantages, whereas dependency on technology and decreasing levels of face-to-face contact are the main disadvantages.

Access to more information and student autonomy are the principal advantages of increasing the use of electronic devices in education. With the internet, students can access all the information available about any topic, regardless of what books and other resources are available in the school. Furthermore, students can focus on whatever topic or subject they want and study it in depth. A prime example of this is the number of online university courses available to students, covering a myriad of subjects that, up until recently, were unavailable to most learners. This has resulted in more people studying third-level degrees than ever before at a pace and schedule that suits them.

The main disadvantages associated with the increasing use of technology in education are the dependency on this technology and the decrease in face-to-face interaction between students. With many students now using the internet as their primary source of information, they often struggle to use other academic resources to find what they’re looking for. As well as this, students spend more time looking at computer screens by themselves than interacting with each other, which is thought to lead to lower levels of emotional intelligence. For instance, the recent explosion in smartphone use has been at the expense of genuine human interaction. This results in soft skills, such as verbal communication and empathy, being affected.

In conclusion, the benefits technology brings to education, such as unrestricted access to information and student autonomy, must be weighed against the drawbacks, such as dependency on this technology and the negative effects on human interaction.

For more detail on how to answer advantage and disadvantage questions, please visit our  advantage and disadvantage lesson . 

Discuss Both Views Question (Discussion Essay)  

agree or disagree essay outline

Discuss both points of view and give your opinion.

Example Question 

Technology is being used more and more in education. Some people say that this is a positive trend, while others argue that it is leading to negative consequences.

Discuss both sides of this argument and then give your own opinion.

1- Paraphrase Question and/or state both viewpoints.

2- Thesis Statement

3- Outline Sentence

1- State first viewpoint

2- Discuss first viewpoint

3- Reason why you agree or disagree with viewpoint

4- Example to support your view

1- State second viewpoint

2- Discuss second viewpoint

Sentence 1- Summary

Sentence 2- State which one is better or more important

There is an ever-increasing use of technology, such as tablets and laptops, in the classroom. It is often argued that this is a positive development, whilst others disagree and think it will lead to adverse ramifications. This essay agrees that an increase in technology is beneficial to students and teachers.

The Internet has provided students with access to more information than ever before. This has allowed learners to research and learn about any subject at the touch of a button. It is therefore agreed that technology is a very worthwhile tool for education. Wikipedia is a prime example, where students can type in any keyword and gain access to in-depth knowledge quickly and easily.

However, many disagree and feel that technology deprives people of real human interaction. Human interaction teaches people valuable skills such as discourse, debate and empathy. Without these soft skills, many people find it difficult to become successful in work and their personal lives. Despite this, human interaction is still possible through the internet, and this essay disagrees that technology should be dismissed for this reason. For instance, Skype and Facebook allow people to interact in ways that were never before possible.

While the benefits of technology, particularly the internet, allow students to tap into limitless sources of information, some still feel that people should be wary of this new phenomenon and not allow it to curb face-to-face interaction. However, as long as we carefully consider the importance of human interaction in education, the educational benefits are clearly positive.

For more detail on how to answer discussion questions please visit our  discussion essay lesson . 

Problem and Solution Questions

agree or disagree essay outline

Problem and solution.

Cause and solution.

Students are becoming more and more reliant on technology.

What are some of the problems associated with reliance on computers, and what are some of the possible solutions?

2- Outline Sentence

1- State Problems

2- Explain First Problem

3- Explain Second Problem

4- Example of Second Problem

1- State Solutions

2- Explain First Solution

3- Explain Second Solution

4- Example of Second Solution

Learners are becoming increasingly dependent on technology, such as the Internet and mobile devices. This essay believes the main problems associated with dependence on computers are the lack of original thought and copying original work from others and suggests critical thinking classes and writing analysis software as the most viable solutions.

The principal problems with over-reliance on technology are people being unable to think for themselves and plagiarism. With access to so much information, students often rely on other people’s opinions instead of forming their own. As well as this, they often use search engines to answer a question and copy the text from a website rather than thinking about the question. This practice is prohibited in schools and universities and stunts students’ intellectual development because they will never truly think for themselves, which is what university is supposed to be for. For example, many teachers complain that students copy web pages straight from Wikipedia word for word rather than giving a reasoned answer to their questions.

Solutions to these worrying problems are special classes to focus on critical thinking and teachers using anti-plagiarism software to detect copying. If teachers create situations where students have to infer meaning and express opinions based on a small amount of information, this will ensure that students have an opportunity to develop these skills. Also, if students know that their assignments are being checked for plagiarism, this will be enough to deter them from doing so. For instance, many universities already use this kind of software to scan coursework for plagiarism, and it could be extended to include all homework by learners in both secondary and tertiary education.

In conclusion, the main problems with the overuse of technology in education are the lack of original thought and plagiarism. These can be solved through special classes that teach students analytical skills and plagiarism detection software.

For more detail on how to answer problem and solution questions please visit our  problem and solution lesson . 

Two-Part Questions

agree or disagree essay outline

There will normally be a statement, and they will then ask you to answer separate questions.

As most people spend a major part of their adult life at work, job satisfaction is an important element of individual wellbeing.

What factor contributes to job satisfaction?

How realistic is the expectation of job satisfaction for all workers?

2- Outline Sentence (mention both questions)

1- Answer first question directly

2- Explain why

3- Further explain

1- Answer second question directly

As most adults spend most of their time at work, being content with your career is a crucial part of a person’s health and happiness. This essay will first suggest fair pay as a key element leading to job satisfaction, and it will then state that it is not very likely that everyone can be happy with their job.

The most important thing that satisfies someone at work is being compensated fairly. If those more senior than you respect you as a person and the job you are doing, then you feel like you are valued. A fair salary and benefits are important marks of respect, and if you feel you are being underpaid, you will either resent your bosses or look for another job. These two factors came top of a recent job satisfaction survey conducted by Monster.com, which found that 72% of people were pleased with their current role if their superiors regularly told them they were appreciated.

With regard to the question of happiness for all workers, I think this is and always will be highly unlikely. The vast majority of people fail to reach their goals and end up working in a post they don’t really care about in return for a salary. This money is just enough to pay their living expenses which often means they are trapped in a cycle of disenchantment. For example, The Times recently reported that 89% of office workers would leave their jobs if they did not need the money.

In conclusion, being satisfied with your trade or profession is an important part of one’s well-being, and respect from one’s colleagues and fair pay can improve your level of happiness; however, job satisfaction for all workers is an unrealistic prospect.

Can I get a band 8 or 9 following these structures? 

Nobody can give you a Task 2 IELTS structure that guarantees high scores. Your score is dependent on how good your grammar and vocabulary are and how well you answer the question. A good structure will help you answer the question to some extent and boost your score for coherence and cohesion, but you must use relevant ideas and use these ideas well to answer the question.

You can see how my student scored a Band 8.5 in IELTS Writing here:

agree or disagree essay outline

Next Steps 

We hope you found those IELTS Writing Task 2 structures useful. Looking for some more sample questions? Here are over 100 sample questions from past exam papers.

If you would personalised feedback and guidance until you get the score you need, you can join the Waiting List for my VIP Course here.

' src=

About Christopher Pell

My name is Christopher Pell and I'm the Managing Director of IELTS Advantage.

I started IELTS Advantage as a simple blog to help 16 students in my class. Several years later, I am very humbled that my VIP Course has been able to help thousands of people around the world to score a Band 7+ in their IELTS tests.

If you need my help with your IELTS preparation, you can send me an email using the contact us page.

  • What can IELTS do for you
  • Ways to take IELTS
  • Who accepts IELTS?
  • Sample test questions
  • IELTS Trial Test
  • Understanding your score
  • Trust IELTS
  • On test day
  • Test centres
  • IELTS One Skill Retake
  • Cancellations, refunds...
  • Access arrangements
  • Getting and sharing...
  • Improving your results
  • Academic Institutions
  • Why accept IELTS?
  • IELTS Scoring
  • Compare IELTS
  • IELTS for your sector
  • Get started with IELTS
  • Verifying IELTS results
  • Research reports
  • Test statistics​
  • Research funding
  • Awards and scholarships
  • Previously funded...
  • News and Insights

Need help finding something? Enter a search term below

IELTS I've got this logo

How to write an agree/disagree essay for IELTS Writing Task 2

Date Published

01 February 2023

This article was first published on WeLoveIELTS.org (this website is now closed)

Knowing how to write an agree or disagree essay is very important because if you get this type of task question in the test and you don't know how to approach it, you might not get a very high score. To what extent do you agree or disagree with this statement?

An agree/disagree question is very similar to the one above. Let’s look at two typical agree/disagree essay questions:

  • Some people believe that nowadays we have too many choices. To what extent do you agree or disagree with this statement?
  • Living in a country where you have to speak a foreign language can cause serious social problems as well as practical problems. To what extent do you agree or disagree with this statement?

Can you see how these are similar to my question at the beginning? Both include a statement (= a sentence expressing an opinion) and ask you to what extent (= how much) you agree or disagree with it.

OK, so what are you going to do first? Start writing? Absolutely not .

After you’ve carefully read the task question and understood the topic, the first thing to do is to ask yourself to what extent you agree or disagree with the statement. There are three possible cases:

  • You agree completely
  • You disagree completely
  • You partially agree (which means you also partially disagree)

Next, ask yourself: ‘ Why do I think that?’ This is a very useful question because by answering it you will start generating ideas that you will then include in your essay.

How many ideas should you come up with? In all three cases the secret is that less is more, so I recommend having no more than four in total.

Let’s have a look at an example from Cambridge IELTS 11:

  • Governments should spend money on railways rather than roads. To what extent do you agree or disagree with this statement?

I’ve read the question and now I’m going to make some notes before I start writing. Here are my notes:

Partially agree - Both needs funding Why?

  • Safety reasons
  • Taxpayers expect government investment in both

Two main ideas. Now we have some direction and know where we are going with our essay. Should you now start writing? Not yet. You’ll need to develop these ideas, and the best way to do this is to give explanations, details, reasons and examples. Let’s add these to the notes.

  • Safety reasons - Both roads and trains are widely used / all ages / need to be kept in good condition / if not, risk of crashes / example: Ponte Morandi collapse 2018 – Italy / lots of casualties / could have been avoided with more funding - maintenance / Trains are crowded at rush hour -> a railroad accident might be a terrible tragedy
  • Taxpayers expect government investment in both - governments need to provide good services / citizens pay tax for this / example: train commuters pay to have efficient trains / if not – disruptions - late for work / same is true for road users / example: opening a new highway -> less traffic

Notice that I didn’t write full sentences but notes. Keep your full sentences for the essay! If you don’t do this brainstorming exercise before you start writing, the risk is that you’ll write whatever comes to your mind, and your essay will probably be confusing to read.

Top Tip: Think of how you’re going to structure your text. Keep life simple and always aim at four paragraphs: introduction, two body paragraphs and conclusion.

Introduction

Two sentences are enough here. In the first sentence you should paraphrase the task question. In the second sentence say if you (partially) agree/disagree so you immediately let the reader know what you think.

Two main paragraphs

Why these paragraphs? A paragraph contains ideas about a single subject and using them will make your essay organised, structured and easy to read. When writing an agree/disagree essay there are, again, three possible options:

  • You completely agree - First paragraph: reasons why you agree. 2nd paragraph: other reasons why you agree.
  • You completely disagree - First paragraph: reasons why you disagree. 2nd paragraph: other reasons why you disagree.
  • You partially agree - First paragraph: reasons why you agree. 2nd paragraph: reasons why you disagree.

Remember: it’s much better to have few well-developed ideas than a lot of poorly developed ones , so when you write the paragraphs make sure to give reasons, examples and details. All these must be relevant to the reason you agree/disagree.

Again, keep life simple and write one or two sentences only. You should briefly repeat and summarise your answer to the question. Don’t introduce information that you didn’t mention in your paragraphs before. We need a conclusion to conclude right? So, don’t introduce other reasons or topics at the end of your text.

  • Read the topic of the question and make sure you understand it
  • Ask yourself if you agree or not with the statement in the question
  • Brainstorm ideas before you start righting
  • State your opinion in the introduction
  • Use four paragraphs
  • Logically organise the main paragraphs (for example, one for reasons why you agree and the other for reasons why you disagree)
  • Extend and develop your ideas with reasons, examples and explanations
  • Write a short conclusion.
  • Start writing immediately
  • Include too many different ideas. Less is more!
  • Introduce more information in the conclusion.

Hope this helps you with your writing. Good luck!

  • Accessibility
  • Legal & policies

2024. IELTS is jointly owned by the British Council; IDP IELTS; and Cambridge University Press & Assessment

An opinion essay

An opinion essay

Learn how to write an opinion essay.

Do the preparation task first. Then read the text and tips and do the exercises.

Preparation

Matching_MjMxMDM=

Some people think that some types of criminals should not go to prison. Instead they should do unpaid work in the community. To what extent do you agree?

Owing to the great variety of crimes that can be punishable by prison, some people argue that not all criminals are the same and it would therefore be more appropriate to give certain criminals community service instead. I agree that in some cases, prison may not be the best solution and community service would probably have more benefits.

One justification given for prisons is to keep society safe by removing criminals from the outside world. So the first thing to consider is if someone who has broken the law is a danger to other people. In the case of violent crime, there is an argument to keep the perpetrator away from society. However, burglary or possession of drugs, for example, does not involve violence against other people so the criminal does not present a direct danger to anyone in the community. Keeping these types of criminals in prison is expensive for the taxpayer and does not appear to be an effective punishment as they often commit the same crime again when they come out of prison.

Personally, I also believe punishments should reform people so they do not reoffend. A further reason not to put these people in prison is that they may mix with more dangerous and violent criminals, potentially committing a worse crime when they are released. By keeping them in the community, helping others, they not only learn new skills, but they could also develop more empathy and care towards others. If this occurs, society can only benefit.

Critics of this more rehabilitative approach to crime believe that justice should be harsh in order to deter people from committing similar crimes and that community service could be less likely to have that effect. However, there is very little evidence to suggest that long prison sentences deter criminals.

In conclusion, putting criminals who are not a danger to society in prison is expensive and, in my opinion, ineffective, both as a deterrent and as a form of rehabilitation. Community service for non-violent crimes benefits both society and the offender. That said, it would be useful to have more data to work out whether community service or prison is more likely to stop someone reoffending. I strongly believe that decisions on how best to deal with criminals should be based on evidence of what actually works.

  • Introduce your essay by restating the question in your own words.
  • If the essay asks you to what extent do you agree?,  make your opinion clear throughout. You can either agree, partially agree or disagree with the statement, explaining and justifying your opinion.
  • Introduction
  • The first reason why you agree/disagree
  • The second reason why you agree/disagree
  • The third reason why you agree/disagree (if you have one)
  • Use phrases to organise and link your ideas, e.g. Owing to … , One justification for … , The first thing to consider is … , A further reason … , In conclusion ... .
  • If you do not have solid evidence for your ideas, use modal verbs such as might , may or could  (e.g. they could  develop more empathy and care ) or other tentative phrases (e.g. it does not appear to be an effective punishment ).
  • Conclude by restating your opinion and summarising your two or three main arguments.

Do you agree that community service is better than prison for some crimes?

Language level

Good day Sir/Madam,

I would like to ask about the expression ‘So the first thing is to…’. In academic and formal writing, should I avoid using ‘thing’ or ‘so’ or is it still fine? Thanks a lot for taking your time to read this. Hope to hear from you.

Your sincerely, Kimmie

  • Log in or register to post comments

Hi kimmie,

Thanks for your question. In academic and formal writing, there is a general preference for precision over vagueness, so generally speaking it's better to avoid this usage of  so  and  thing  and use more precise and academic terms when they are available (e.g. Therefore, the first factor/issue to consider is ... ).

However, if you search academic writing, you will find writers do use  thing  sometimes, especially in fixed phrases (e.g.  the first thing /   the same thing ) or technical terms ( e.g. the Internet of Things ). Some writers might also use more general terms first before moving onto more precise terms.

(Note: The essay on the page above is in a somewhat formal style but not very formal.)

I hope that helps.

LearnEnglish team

Hi Jonathan

This is immensely valuable for me. Thank you so much for your swift response. Hope you have a nice day.

In my country it is often debated whether community services are better than prison for certain crimes. I think it's pretty obvious that it should be an option for less important faults. However, sometimes it happens that a judge decides to grant this benefit to some type of powerful criminals who cause more significant damage to society than others who do not have the same possibility, such as financial criminals or corrupt government agents. In my opinion, these types of sentences can have a bad impact on people, since they see that these behaviors are not severely punished and are somehow protected by the law. In conclusion, I agree that certain types of crimes should avoid prison, but I believe that before implementing it, the society must agree on which crimes will be able to enjoy this possibility to ensure that justice is fair for all.

please approve for testing

Hello aaron,

We check all comments before publishing them, which is why it's taken some time for your comment to be published.

Normally we delete a comment like this -- and we deleted the other one you made -- but have let this one through so you can see that it works. 

We are a small team, so it can sometimes be several hours before comments are published -- just wanted to tell you for the future.

We look forward to hearing more from you!

All the best, Kirk LearnEnglish team

The subject of how criminals should be punished is complicated, there are a lot of different opinions defending their points of view. There are two main ideas the first is arresting all the offenders, and the second idea is separating those not dangerous to society, and sending them to work in the community. In my opinion, I agree with the second idea, because there are many expenses with the prisons and the prisoners, so if lessen the population in this environment the conditions and the quality could be better than if there were overpopulation, avoiding justice being done right. Furthermore, these people aren't violent and dangerous to others, so with bad company in the prison, they might become bad people too. In this case, it's an awful idea to mix a different kind of person. Therefore, it's an excellent suggestion to oblige these people to do work in society, but with supervision in the right way and strictly.

On account of the differences between several types of crimes, not all criminals should be sent to prison. It is suggested that giving certain offenders community service would be more appropriate than just jailing them.

Honestly, I completely agree that the right way to protect people from harm is to imprison criminals who conduct violent crimes that endanger human physical health. Nevertheless, this penalty may be ineffective with offenders who have administrative guilts or non-violent offenses. Since they often commit the same crime after getting out of prison, this is just a temporary punishment to ensure the security of citizens for a certain period. The nature of the problem is these crimes originate from greed and bad habits which lead to their deviant behaviors but have not yet resulted in such severe consequences that affect human life.

From my perspective, the government would rather reform these types of offenders than deter them as a kind of punishment. It is better to have an alternative that perceptively impacts these people’s awareness which is to make them do community service. Jailing these criminals just increases their hatred and develops antisocial personality. By keeping and controlling them in a community, besides learning new skills, they amend their bad manner as well as develop more empathy and care towards others.

Given these points, the application of punishment should be based on the nature of each specific crime. Prisons should only be used for perpetrators committing directly dangerous crimes to people's lives. Making non-violent criminals do community service is more of a practical way to optimize tax wasted for prisons as well as give them a chance to fix their wrong behaviors.

There are several types of crimes and for each of them, a different sentence is dictated. A paramount question to be considered is whether community servers are more accurate than prisons for some crimes.

To begin with, community servers would help and support criminals through specialized psychologists. Not only would community servers help and support them, but also it would give them jobs to help them give the society back a contribution in return.

In addition, prisons only contribute to worsening the situation owing to the fact that criminals will commit crimes again. The key to excellent behavior is learning. If they learn suitable habits, they will adapt to society and help it.

Furthermore, it is necessary to have empathy with criminals and not judge them because their hate increases when they are condemned in prison. A further reason for this is that hate feeds hate. Nevertheless, some criminals must be in jail due to their danger on the streets. It is difficult to know that a person who kills someone else, is going to change even if he/she receives help from psychologists.

In contrast, most of society believes that all criminals should be in prison. However, prisons must be all comfortable like a hotel in order to bring for the prisoners a place to recover themselves. The reason why they must live in appropriate conditions is due to his mental health. For instance, in Norway, prisons are like hotels because the government considers that criminals need a second chance to live in a community

To conclude, it would be suitable for all criminals to live in accurate prisons in order to recover themselves. Despite the cost that kind of prisons are supposed to take, they are worth paying for them. In my opinion, safety is priceless and is more important than the cost.

Online courses

Footer:Live classes

Group and one-to-one classes with expert teachers.

Footer:Self-study

Learn English in your own time, at your own pace.

Footer:Personalised Tutor

One-to-one sessions focused on a personal plan.

Footer:IELTS preparation

Get the score you need with private and group classes.  

IELTS  TOEFL  PTE   YDS  YÖKDİL  iTEP

  • YDS – YÖKDİL
  • AUDIO BOOKS

IELTS ESSAY TEMPLATE-agree – disagree

IELTS ESSAY TEMPLATE-agree – disagree

INTRODUCTION

To agree or disagree with the statement that (summary of the question) is  an important issue. Putting the discussion in a wider context, ( a few words about the subject ) has always been debatable. Even though some people think that (the idea which you dont like) , I wholeheartedly believe that ( the idea which you like ) . First I will discuss some arguments supporting my ideas about this statement, after which some aspects against that will be presented.

 On the one hand, many people agree with this statement for many noteworthy reasons. The most remarkable is that  (reason 1………….).For instance, (example). Therefore , ………  Another key reason is that (reason 2……..) .  For example, (example). Consequently, …………

On the other hand, other people disagree with this statement for many reasons.  The most important is that (reason 1……..) .As an example, (example).T herefore , ……….  One more reason to disagree with the statement is that (reason 2……) .The best example (example). Consequently ,…………

  All in all, when all the specific reasons and relevant examples are considered and evaluated,  I strongly  agree with the idea supporting this statement because its benefits outweigh its drawbacks.

[ Cancel replying ]

Save my name, email, and website in this browser for the next time I comment.

' src=

thanks ,you solve my problems

agree or disagree essay outline

Education – IELTS Agree/Disagree Essay Sample

In this post, we will look at an agree/disagree essay example from the IELTS writing task 2 test. Students often ask if the questions are repeated year after year and the answer is no, but the topics are. There are so many questions written each year, you may find your practice answering various questions on different topics. For example, you could write essays to answer questions about education or the environment, which benefits you because you learn vocabulary associated with those topics and develop ideas that can help you in your writing test.

Practicing writing IELTS task 2 essays  on a range of topics is a great way to learn new vocabulary for those topics, but also to practice your essay structures . You begin to develop your ideas around those topics, thinking of examples and giving your opinions. 

If you would like to purchase a 31  page PDF download that is easy to read and print out please take a look at the bookshop >

If you would like to learn how to structure an agree/disagree essay please click the button below >

How to write an Agree/Disagree Essay

Take a look at the IELTS essay example below >>

Over the years, many people considered learning to read, being able to write and having knowledge of solving mathematical problems as the foundations of education, however, some people believe that learning to use a computer should be added to the said list. In my opinion, I strongly agree that it would be very beneficial for all to include computer skills in the framework of our education. Firstly, this essay will discuss that with the advancement of our technology it is inevitable that everyone should learn how to use a computer. Secondly, it will discuss the fact that almost all jobs nowadays involve the use of technology.  

On the one hand, computers have become a huge part of our everyday lives as almost everything can be done with just one click of a button. Having skills in computing can help a student to navigate the Internet and that aids them in researching answers for their homework. Computers can replace thousands of books and this eases the burden of bringing a lot of books to school. For example, in the past, if you needed school supplies, you would go to a supermarket or a department store but now, just by using your computer or cell phone you may opt to go shopping online in the convenience of your home, by just using apps like LAZADA and OLX.

On the other hand, almost every job needs a computer in one way or another in order to complete everyday tasks. Therefore, knowing how to use a computer or tablet is a basic skill that most workplaces will require their employees to have. Computers are also used to perform groundbreaking services in some workplaces, like hospitals and private clinics. For instance, doctors use this technology so that they can perform surgeries and get accurate results. LASIX technology has helped thousands of people to bring their vision back to normal, this would be near to impossible without the use of highly advanced computer programs.

In conclusion, adding computer skills into our education system is vital towards the road of advancement in technology. I strongly agree that learning skills like this will benefit not only the individual but also the progress of any country.

(Word count – 365 / Band score 8)

IELTS Writing Task 2 Question Types

Agree/Disagree Essays ADVANTAGE / DISADVANTAGE ESSAY CAUSE AND EFFECT ESSAY IELTS Problem/Solution Essay IELTS Discussion Essay  POSITIVE OR NEGATIVE ESSAY Direct Question Essay IELTS Writing Task 2 Student Essay

Feedback

  • Task Achievement  – The essay provides an answer to the question asked, supported by relevant examples.
  • Coherence and Cohesion  – The answer has been divided into clear logical paragraphs and each main body paragraph only has one main idea. There are cohesive links between the main body paragraphs .
  • Lexical Resource  – There is evidence of a wide range of vocabulary, with no errors in the text.
  • Grammatical Range and Accuracy  – The answer has no grammatical errors. The sentences have a wide range of structures. 

We hope you found this post useful in helping you to study for the IELTS Test . If you have any questions please let us know in the comments below or on the Facebook page.

The best way to keep up to date with posts like this is to like us on Facebook , then follow us on Instagram and Pinterest . If you need help preparing for the IELTS Test, join the IELTS Achieve Academy and see how we can assist you to achieve your desired band score. We offer an essay correction service, mock exams and online courses.

Related Posts

IELTS Agree/Disagree Essay Sample 8 - Government

IELTS Cause/Effect Essay Sample 1 – Crime

IELTS Writing Task 2 cause/effect essay example that is a band score 8. The question…

nstant Grammar Checker - Correct all grammar errors and enhance your writing.

How To Write An Advantages Or Disadvantages Essay

Do you want to learn how to create an advantages/disadvantages essay worthy of a high…

1 thought on “Education – IELTS Agree/Disagree Essay Sample”

agree or disagree essay outline

Hi! Can you please suggest me the structure to write opinion essay with partially agree and partially disagree. can we write one body for agree and one for disagree. I would really appreciate if i can get one sample upon this. Thanks& Regards.

Leave a Comment Cancel Reply

Your email address will not be published. Required fields are marked *

Save my name, email, and website in this browser for the next time I comment.

Preparation for the IELTS Exam

How to write an IELTS thesis statement.

Ielts essay introductions and thesis statements..

updated: July 27th 2022. When writing the introduction of an IELTS essay there are two steps that need to be taken. Paraphrase the task question and write a Thesis Statement . If the question asks for an opinion then it must be in the thesis statement. It depends on the type of essay you are writing as they are not all the same. It is advisable to write a thesis in the introduction for every type of essay. A good thesis statement can help you get a good band score in task response in the writing section.

The thesis statement tells the examiner what the essay is going to be about and the conclusion of the essay paraphrases the thesis statement. Some teachers have other views and say that a thesis statement is not needed. There are different approaches to writing an IELTS essay. If you have 10 IELTS teachers in a room there will most likely be 10 different opinions on the matter.

In my opinion, a thesis statement is logical and shows the reader what to expect in the rest of the essay. However, do not confuse Thesis statements with memorised outline sentences. See this link here about that.

What exactly is a thesis statement?

‘A thesis statement tells the reader what your essay is going to be about in one or two sentences. It usually includes your opinion or states your position’
  • Don’t use rhetorical questions in the thesis. The thesis statement is not a question.
  • Make it clear what you are going to write about briefly (1 or 2 sentences)
  • If it’s an opinion essay, give 2 reasons for your opinion in the thesis statement.
  • The thesis statement comes after paraphrasing the question.
  • Do not write an outline sentence, these look like a cliche and memorised. Examiners are trained to spot memorised phrases, for instance: ‘ This essay would like to explore reasons for this in more detail’
  • Don’t confuse thesis statements with the above outline sentence. Outline sentences are for very long academic essays. See this lesson here on phrases to avoid.

For each essay type this is what should be in the Thesis statement:

1. Opinion essays: write 2 reasons for your opinion. 2. Advantage disadvantage essays:  state the advantage and the disadvantage, 3. Problem solution essays:   briefly state 1 or 2 problems and possible solutions. 4 . Discussion essays: after paraphrasing both sides of the argument, give your opinion with a reason why you hold that view. 5. Two part question essays: answer the 1st question then answer the 2nd question briefly.

First you have to identify what kind of essay it is.

The first step before you begin to write is to make sure you understand the question, then identify what kind of essay this will be. There are 5 variations on an IELTS discursive essay click here to see a lesson on this .

Before you write the thesis statement you will need to paraphrase the question , click here for a lesson on this. Click here to see how to write a good introduction to a Problem Solution essay.

1. Discussion essay.

agree or disagree essay outline

This is the Paraphrased introduction with a Thesis statement.

A number of people believe serious crimes need to have a set punishment, whereas others argue that the situation of the crime must be considered. I agree that the circumstances of the crime itself should be taken into account because every case is different.

Thesis Statement: ‘ I agree that the circumstances of the crime itself should be taken into account because every case is different’

It is important to give a brief reason for your view, the body paragraphs should expand on this. The examiner can clearly see your position in the introduction. In this essay, my position is that every case is different and a prison sentence would not be appropriate.

Note: For a more academic feel, rather than using a personal pronoun to state your opinion, you could state your position like this below:

This essay agrees that the circumstances of the crime itself should be taken into account because every case is different.

2. Opinon essay.

agree or disagree essay outline

Again you need to paraphrase the question and then clearly agree or disagree , remember to choose just one side.. the words: ‘To what extent do you agree or disagree’ means how much do you agree/disagree, or how far do you agree/disagree. You should mention the other side of the argument but stick to your own opinion. Balanced essays do not necessarily get a higher band score.

Give 2 reasons for your view in the thesis statement here. This is the Paraphrased introduction with a Thesis statement:

It is argued that society would be better off if every type of advertising was prohibited. I disagree that all advertising should be disallowed as this policy would not benefit society and would negatively impact the economy.

Thesis statement: ‘ I disagree that all advertising should be disallowed as this policy would not benefit societ y and would negatively impact the economy.’

In this thesis statement I have clearly stated why I hold the opinion that advertising should not be banned with 2 reasons (no benefit to society and bad for the economy)  Remember to keep the introduction to under 55 words or it will be too long.

As mentioned before, you can use a more academic way to state your view, such as:

This essay disagrees that all advertising should be disallowed because this policy would not benefit society and would have a negative impact on the economy.

3. Advantages disadvantages essay.

agree or disagree essay outline

This is the Paraphrased introduction with a Thesis statement:

In recent times, people can reside wherever they want in the world because of the progress that has made in technology and transport. The main advantage is the career opportunities that a person can get outside their own country, while a possible downside would be the stress of adjusting to living in a different culture.

Thesis statement: ‘ The main advantage is the career opportunities that a person can get outside their own country, while a possible downside would be the stress of adjusting to living in a different culture.’

Try keeping the whole introduction under 55 words as you don’t want it to look like a body paragraph. You can write about 2 advantages and 2 disadvantages, but you need to keep the introduction concise.

Another method is to just refer to the advantage and state the disadvantage, such as:

In recent times, people can reside wherever they want in the world because of the progress that has made in technology and transport. Although there are advantages, the downside would be the stress of adjusting to living in a different culture.’

This allows you to keep the thesis statement concise especially if you want to cover two advantages (or disadvantages)

4. Advantages disadvantages outweigh essay (this needs your opinion).

agree or disagree essay outline

This type of advantage disadvantage essay is special because it asks ‘ Do the advantages of this outweigh the disadvantages?’. This means you have to write about what side you think is stronger and reflect that in the essay and also in the thesis statement. You need to state an opinion here.

Some experts argue that children should study a new language at primary school as opposed to secondary school. This essay agrees that the advantages are stronger than the disadvantages because youngsters pick up and master new languages much easier than at a high school age.

Thesis statement: ‘This essay agrees that the advantages are stronger than the disadvantages because youngsters pick up and master new languages much easier than at a high school age .’

I have underlined the reason for my opinion here. Giving a reason for your opinion is important in a thesis statement. The introduction is 44 words long so that will be fine. Remember to also address the other side of the issue in the main body paragraphs, the side you think is weaker.

Another method here is to use this style which is very concise at 32 words but it does not contain a reason for my view.

Some experts argue that youngsters should study a new language at primary school as opposed to secondary school. In my view, the advantages of younger children learning new languages outweigh the disadvantages.

5. Problem solution / causes solution essay.

ielts problem solution essay

People living in large cities have to deal with many issues in their day to day lives. The main problems people face are high rental costs and overcrowding. Some possible solutions would be to build more affordable housing and more green spaces.

Thesis statement: ‘ The main problems people face are high rental costs and overcrowding. Some possible solutions would be to build more affordable housing and more green spaces.’

I have listed 2 problems high rents, overcrowding and 2 solutions affordable housing, green spaces . You can choose just one problem and one solution and that would be fine.

6. Two part question (direct question essay).

direct question essay ielts

The world wide web is a huge source of knowledge which has created opportunities for people worldwide to study. However, not all information on the internet can be trusted, so the government needs to put measures in place to protect people from false information.

Thesis statement: ‘ However, not all information on the internet can be trusted, so the government needs to put measures in place to protect people from false information.’

Two part question essays are sometimes called ‘Direct question’ essays. They consist of 2 direct questions in the task question. Sometimes they will ask for your opinion such as: Do you think….?  or   What do you think…?

If the question asks for the opinion then you must state it. You should briefly answer the 2 questions in the thesis statement then give more detail in the body paragraphs.

Check out the blog posts about how to identify the 5 essay types and also how to paraphrase the question . These are key stages before writing your thesis statement. Make sure your whole introduction is under 55 words or it will be too long.

Take a look here at how to write a good introduction in writing task 2. Now you can try.

Here is a Discussion essay question where you have to give your opinion. Can you write a Thesis statement for it?

Some people believe that the best way to deal with heavy traffic in city centres is for privately owned vehicles to be banned, others however think this is not a realistic solution.  Discuss both sides and give your own opinion.

Task question again:

Some people believe that the best way to deal with heavy traffic in city centres is for privately owned vehicles to be banned, others however think this is not a realistic solution. Discuss both sides and give your own opinion.

Paraphrased introduction: Some people argue that prohibiting private cars from city centres is the best way to tackle traffic congestion, whereas others say that this is unrealistic.

Thesis statement: I agree that private vehicles should be banned from city centres and more investment needs to be put into public transportation to alleviate traffic jams.

Full introduction with thesis statement:

Some people argue that prohibiting private cars from city centres is the best way to tackle traffic congestion, whereas others say that this is unrealistic. I agree that private vehicles should be banned from city centres and more investment needs to be put into public transportation to alleviate traffic jams.

Leave a comment below if you have any questions..

Leave a comment cancel reply.

👋 Nhắn cho DOL để tìm hiểu chi tiết về các khóa học IELTS nhé!

Cách viết IELTS Writing Task 2 dạng Agree or Disagree chi tiết

Dạng bài viết Agree-Disagree Essay (hay còn gọi là Opinion Essay) là 1 trong 4 dạng câu hỏi chính mà bạn sẽ gặp khi thi ielts writing task 2 agree or disagree . Mật độ của dạng đề này tương đối cao nên bạn cần có một phương pháp làm bài cụ thể cho dạng đề này một cách chính xác và hiệu quả để vừa tránh mất thời gian vừa để đạt được điểm cao.

DOL IELTS Đình Lực

1. Khái quát về dạng bài Agree or Disagree trong IELTS Writing Task 2

1.1 yêu cầu chung.

Đề bài sẽ đưa ra cho bạn một vấn đề xã hội phổ biến mà bạn nên biết. Từ đó bạn sẽ phải lựa chọn đồng ý/không đồng ý với vấn đề hoặc chỉ đồng ý một phần.

Bạn sẽ phải đưa ra bằng chính luận điểm và lập luận cùng với những ví dụ từ chính bản thân mình. Câu trả lời của bạn phải đáp ứng được 4 nhu cầu sau để có thể đặt được điểm cao:

Task Response: Bạn phải nắm rõ tất cả các khía cạnh của chủ đề và câu trả lời phải liên quan trực tiếp đến câu hỏi. Đoạn văn phải tối thiểu 250 từ.

Coherence and Cohesion: Bài viết của bạn phải có sự gắn kết: Bạn phải biết cách sắp xếp toàn bộ lập luận và nội dung của một bài luận theo trật tự logic nhất định. Ngoài ra, câu trả lời của bạn còn phải dễ hiểu.

Lexical Resource:Không chỉ biết cách dùng từ đúng, bạn cần có sự phong phú, tự nhiên cũng như thể hiện được tính linh hoạt trong việc lựa chọn từ ngữ của người viết.

Grammatical Range and Accuracy:Khả năng sử dụng các cấu trúc câu đơn, câu ghép, câu phức thành thạo và chính xác cũng là yêu cầu của bài viết.

agree or disagree essay outline

1.2 Cách nhận biết

Cách nhận biết dạng agree-disagree tương đối đơn giản. Sau khi đưa ra vấn đề, dạng đề agree-disagree thường có câu hỏi sau: “ To what extent do you agree or disagree?”

2. Chiến lược viết bài

agree or disagree essay outline

2.1 Trước khi viết

Để có thể giúp các bạn hình dung các bước viết bài một cách cụ thể hơn, chúng ta sẽ cùng thông qua một đề bài IELTS Writing Task 2 agree or disagree sau đây:

Some people think that a huge amount of time and money is spent on the protection of wild animals, and that this money could be better spent on the human population. To what extent do you agree or disagree with this opinion?

video-thumbnail

Hướng dẫn làm bài IELTS Writing Task 2 dạng Agree or Disagree

Phân tích đề bài và xác định câu trả lời

Đối với dạng đề agree-disagree, bạn sẽ có 2 cách để trả lời:

One-sided answer: Bạn đồng ý hoặc không đồng ý hoàn toàn với vấn đề đưa ra

Balance answer: Bản chỉ đồng ý một phần của vấn đề

Ngay sau khi đọc kỹ đề bài, bạn bắt buộc phải có trong đầu câu trả lời cho mình để từ đó xác định hướng viết bài một cách nhanh chóng.

Khi bạn làm dạng câu hỏi này, trước tiên hãy quyết định quan điểm của bạn. Có 2 dạng quan điểm được khuyến khích để viết một câu trả lời tốt là đồng ý hoặc không đồng ý 100% và 50% đồng ý, 50% không đồng ý. Quan điểm của bạn nên được trình bày rõ ràng trong phần giới thiệu của bạn và mở rộng trong các đoạn nội dung

Có một lưu ý và cũng là một bí quyết giúp các bạn có thể xác định hướng viết nhanh là IELTS là một kỳ thi đánh giá khả năng sử dụng tiếng Anh chứ không phải là bài thi kiến thức xã hội. Bạn không cần phải trả lời đúng với những gì bạn nghĩ.

Thay vào đó, hãy chọn câu trả lời giúp bạn dễ viết nhất. Hãy chọn câu trả lời mà bạn có thể đưa ra được luận điểm, lập luận và ví dụ ngay lập tức. Ngay sau khi tìm được cho bản thân hướng viết bài, các bạn hãy tiếp tục sáng bước 2

agree or disagree essay outline

Nhiều bạn cho rằng việc lập dàn ý là không cần thiết và chỉ mất thời gian, thay vào đó nên tập trung viết bài. Cách suy nghĩ này không sai, bài viết là phần duy nhất được chấm điểm. Tuy nhiên, nếu không có một dàn ý, một sườn bài rõ ràng sẽ mang lại rất nhiều hậu quả.

Nghĩ gì viết đấy, các ý tưởng dễ bị trùng lặp do không được sắp xếp có hệ thống. Viết không theo một hướng đi nào rất dễ làm bạn viết lang mang, từ đó có thể dẫn đến lạc đề và bị trừ điểm.

Việc làm dàn ý, nếu thông qua luyện tập thường xuyên, sẽ chỉ mất bạn 5 phút. Các ý tưởng của bạn sẽ được sắp xếp một cách hệ thống. Từ đó, bạn chỉ việc viết theo sườn bài thì bài viết sẽ được hoàn thành nhanh chóng hơn và hiệu quả hơn:

Khi áp dụng Linearthinking để nghĩ và phát triển ý, ta có dàn ý hoàn chỉnh như sau:

Đoạn 1: More and more people read news on the Internet

Internet is convenient => can read anywhere + information is updated quickly + do not cost much => attract more and more people (Internet thuận tiện => có thể đọc bất cứ khi nào + thông tin được cập nhật nhanh chóng + không tốn nhiều tiền => thu hút nhiều và nhiều người hơn)

Internet news is more interactive => people can watch videos + leave comments or join discussions => Example: football news are often accompanied by videos of important moments in the game + a comment section for fans to discuss (tin trên Internet có tính tương tác hơn => có thể xem videos, để lại bình luận hay tham gia bàn luận => Ví dụ: tin tức bóng đá thường theo kèm videos của những khoảnh khắc quan trọng trong trận bóng và phần bình luận cho người hâm mộ)

Đoạn 2: Newspaper still remain the most important source 

2.2 Trong khi viết

Sau khi thiết lập một hệ thống dàn ý cho bài viết của mình, bạn bắt đầu bắt tay vào phần viết bài

Phần mở bài của dạng IELTS Writing Task 2 agree or disagree bao gồm 2 phần chính:

Phần 1 – Giới thiệu chủ đề của bài viết

Bạn phải mở đầu bài viết của mình bằng cách giới thiệu cho người đọc biết chủ đề bài viết của mình. Cách hiệu quả và nhanh chóng nhất để viết câu chủ đề chính là paraphrase lại câu hỏi trong đề bài.

Cấu trúc câu thường dùng:

It is a common belief that S + V

Many people hold the view that S + V

Phần 2 – Câu trả lời

Sau khi giới thiệu chủ đề của bài viết, chúng ta sẽ trực tiếp trả lời câu hỏi của đề bài. Bạn không thể nào vừa hoàn toàn đồng ý và không đồng ý với vấn đề được đặt ra trong bài. Bạn có thể đồng ý một phần và đưa ra luận điểm tại sao bạn không đồng ý phần còn lại. Đừng tự phản bác với câu trả lời của mình ở phần mở bài. Vì thế, việc lập dàn ý là rất quan trọng để đảm bảo bạn có một hệ thống để dựa vào viết bài.

Cấu trúc câu thường dùng cho balance answer:

While I think S + V, I agree/disagree with the view that + S + V.

Cấu trúc câu thường dùng cho one-sided answer:

From my perspective/ In my opinion, I completely agree/ disagree with this view for several reasons.

⇒ Áp dụng cho đề bài này, ta có đoạn mở bài như sau:

There can be little doubt that in recent decades a lot of time and money has been spent on protecting wild animals. While I think spending on protecting these kinds of wild animals can be sometimes beneficial to human, I agree that it is of higher importance to allocate these resources to the human population.

Nhắc lại, với mỗi ý mà chúng ta đưa ra thì nên được viết vào một đoạn riêng. Mỗi đoạn thân bài nên bao gồm 3 phần:

Phần 1: Topic Sentence – Giới thiệu luận điểm

On the one hand, it is true/reasonable that S + V to a certain extent.

Phần 2: Explanation – Giải thích luận điểm

First/To begin with, S + V ⇒ This leads to/Therefore + S + V ⇒ In addition, S + V

Phần 3: Example – Ví dụ

⇒ Ta có ví dụ cho phần thân bài như sau:

Đoạn 1: On the one hand, it is reasonable to provide financial support for wildlife protections. In fact, by saving wild animals, humans are saving themselves. To begin with, wild animals may hold the keys to certain medical breakthroughs, just as many of modern medicines are based on compounds found in plants. Moreover, humans, animals and other forms of life all form a giant ecosystem whereby each part can influence the others. If a certain animal becomes less common, this, in turn, will reduce the population of the animals that prey on it, which is more likely to disrupt the ecological balance and eventually threatens the survival of human society.

Đoạn 2: However, it is sometimes difficult for people to accept that these perfectly legitimate reasons outweigh the immediate requirements of the human population. If money is not available to people who have problems such as mental illness, physical disabilities or to those living in dire poverty, it is inevitable that some will suggest spending less on wild animals. There is the strong argument that by spending more on the education of humans, we could make the world a better place for us and for wild animals.

Ở phần kết bài của dạng IELTS Writing Task 2 agree or disagree , chúng ta tóm tắt lại câu trả lời của bản thân. Lưu ý, các bạn không được phép đưa thêm ý kiến mới vào trong thân bài và cũng không được copy y chang mở bài vào trong thân bài.

Cấu trúc câu thường gặp:

In conclusion, while I support the view that S+ V, I disagree that S+ V.

⇒ Ta có kết bài sau:

In conclusion, while I tend to agree with those who claim that too much money is currently being spent on wild animals at the cost of humans, I disagree that we should cut all funding of wildlife projects, since many of these projects can directly benefit humans. However, a key argument for me is that by spending money on people, we can improve the world generally.

2.3 Sau khi viết

Hãy dùng những phút còn lại sau khi hoàn thành bài viết để đọc lại và phát hiện cũng như sửa chữa các lỗi sai chính tả, ngữ pháp. Sau cho cùng, phần ngữ pháp và từ vựng chiếm 50% số điểm của bạn.

⇒ Ví dụ: Ở những đoạn ví dụ phía trên có một số lỗi như sau:

There can be little doubt that in recent decades a lot of time and money have been spent on protecting wild animals.

Time and money: danh từ không đếm được ⇒ “have” sai ⇒ sửa lại: has been spent.

In fact, by save wild animals, humans are saving themselves.

By save wild animals ⇒ by saving wild animals.

If a certain animal become less common, this, in turn, will reduce the population of the animals that prey on it, which is more likely to disrupt the ecological balance and eventually threatens the survival of human society.

Become ⇒ becomes

agree or disagree essay outline

3. Một số lưu ý khi viết bài

Bài thi Writing Task 2 agree or disagree trong IELTS yêu cầu bạn viết tối thiểu 250 từ, nên hãy lưu ý viết đủ hoặc hơn để không bị trừ điểm.

Bạn nên thể hiện rõ lập trường của mình ở phần Mở bài hoặc phần Kết bài của bài viết IELTS Writing Task 2 agree or disagree thì bài viết của bạn sẽ có tính thuyết phục hơn.

4. Một số bài mẫu

agree or disagree essay outline

5. Giải đáp các câu hỏi thường gặp

Gợi ý cấu trúc bài luận IELTS Writing Task 2 Agree Or Disagree

Mở bài: Giới thiệu sơ qua vấn đề và ý kiến đồng ý/ không đồng ý của bạn

Thân bài: Chia làm 2 đoạn. Trong đó đoạn 1 nói về lý do tiên quyết khiến bạn đồng ý/ không đồng ý. Và đoạn 2 tiếp tục đưa ra những lý do khác để củng cố quan điểm đồng ý/ không đồng ý của bạn.

Kết bài: Tóm lược vấn đề một cách ngắn gọn, ấn tượng.

Có thể sử dụng ngôi thứ nhất trong bài luận IELTS Writing Task 2 Agree Or Disagree không?

Hoàn toàn có thể! Thí sinh được phép sử dụng ngôi thứ nhất để đưa ra quan điểm trong bài luận IELTS Writing Task 2 Agree Or Disagree của mình. Ngược lại, nếu thí sinh không muốn sử dụng ngôi thứ nhất mà thay vào đó là dùng ngôi thứ 3 thì cũng không sao.

Một số đề bài thường gặp trong IELTS Writing Task 2 Agree Or Disagree

“Modern life is increasingly chaotic.” Do you agree or disagree?

“The death penalty is barbaric and should not be legal anywhere.” Do you agree or disagree?

“Libraries are irrelevant in the age of the internet and should not be publicly funded.” Do you agree or disagree?

“The best way to solve the world’s environmental problems is to increase the fuel cost.” Do you agree or disagree?

Những sự nhầm lẫn điển hình khi làm IELTS Writing Task 2 Agree Or Disagree mà thí sinh cần chú ý là gì?

Nhầm lẫn giữa Good và Best

Nhầm lẫn giữa Better và Best

Nhầm lẫn giữa Explode và Explore

Nhầm lẫn giữa Experience và Experiment

Nhầm lẫn giữa Through và Thorough

Nhầm lẫn giữa Angel và Angle

Nhầm lẫn giữa Dessert và Desert

Nhầm lẫn giữa Later và Latter

Nhầm lẫn giữa Affect và Effect

Nhầm lẫn giữa Elude và Allude

Nhầm lẫn giữa Formerly và Formally

Nhầm lẫn giữa Cite, Site và Sight

Nhầm lẫn giữa Principal và Principle

Nhầm lẫn giữa Chose và Choose

Nhầm lẫn giữa Quite và Quite

Bài viết trên đã chỉ ra những dạng câu hỏi IELTS Writing Task 2 agree or disagree , những lỗi thường gặp khi làm dạng bài này cũng hướng dẫn các bạn cách làm dạng bài này hiệu quả với Linearthinking. DOL hy vọng đây sẽ là nguồn tài liệu tham khảo hữu ích cho các bạn trong quá trình ôn thi IELTS. Nếu muốn tìm hiểu thêm cách viết dạng bài này, đừng quên khám phá kho sample essay Task 2 siêu to khổng lồ của DOL nhé!

Bài viết khác

Bộ đề thi ielts writing 2023 thực tế tổng hợp đầy đủ kèm bài mẫu.

Chuẩn bị cho kỳ thi IELTS, đặc biệt là phần thi Writing, luôn là nỗi lo của nhiều bạn học. Để tự tin chinh phục phần thi này, bạn cần nắm rõ cấu trúc đề thi, các dạng câu hỏi thường gặp và có sự luyện tập thường xuyên. Hiểu được nhu cầu đó, DOL đã tổng hợp và biên soạn bộ đề thi IELTS Writing 2023 chính thức tại IDP & BC kèm bài mẫu. Bộ đề này sẽ giúp bạn hiểu rõ cấu trúc đề thi và các dạng câu hỏi thường gặp, luyện tập trả lời các câu hỏi theo hướng dẫn của giám khảo và tăng khả năng phản xạ và tự tin trong phòng thi. Hãy cùng DOL tìm hiểu chi tiết bộ đề IELTS Writing 2023!

đề thi ielts writing 2023 thực tế

Top 10 website sửa bài IELTS Writing uy tín và chất lượng nhất

Trong hành trình học IELTS Writing, việc đạt được band điểm mơ ước đòi hỏi sự nỗ lực và kiên nhẫn từ các thí sinh. Tuy nhiên, nhiều thí sinh vẫn gặp khó khăn trong việc tiến bộ, và lý do chính là thiếu việc luyện tập viết IELTS Writing thường xuyên và không tuân thủ thời gian đủ cho cả hai bài task 1 và 2. Đặc biệt, việc không có người sửa bài Writing thường xuyên cũng là một trong những nguyên nhân chính. Tất nhiên, việc đến trung tâm để có giáo viên chỉnh sửa cũng là một giải pháp hiệu quả. Tuy nhiên, nếu không có đủ thời gian và điều kiện, việc sử dụng công cụ AI sẽ là một lựa chọn hữu ích để rèn luyện và cải thiện kỹ năng viết IELTS của bạn. DOL gợi ý cho bạn 10 website sửa bài IELTS Writing sử dụng công nghệ AI uy tín và chất lượng nhất. 1. Grammarly 2. Write and Improve with Cambridge 3. PaperRater 4. Test Big 5. Google Bard 6. ChatGPT 7. IELTS Answers 8. IELTS Advantage 9. IELTS-Blog 10. IELTS Podcast Hãy cùng DOL điểm qua các website sửa bài IELTS Writing sau nhé!

website sửa bài ielts writing

Cách sử dụng Notion (AI) để tự học IELTS Writing hiệu quả

Notion là một phần mềm ghi chú và quản lý công việc thông minh. Khó ai có thể tóm tắt và giải thích ngắn gọn được Notion là gì bởi vì các tính năng sắp xếp thông tin mạnh mẽ của nó đi cùng với giao diện tối giản và ý nghĩa sử dụng to lớn trong nhiều lĩnh vực cuộc sống như làm việc, học tập, lên ý tưởng, outline, lên kế hoạch,... Trong bài viết này, DOL sẽ giới thiệu qua Notion, cung cấp các tính năng, hướng dẫn cụ thể cách sử dụng Notion (AI) để tự học IELTS Writing hiệu quả và gợi ý những mẫu template tiện lợi cho những ai mới bắt đầu sử dụng Notion như một trợ thủ đắc lực trong quá trình ôn luyện IELTS Writing.

cách sử dụng notion ai để tự học ielts writing

Bộ sách Tổng hợp Bài mẫu IELTS Writing Sample 2023

DOL IELTS Đình Lực dành tặng miễn phí bộ sách TỔNG HỢP BÀI MẪU IELTS WRITING SAMPLE 2023. Tải về ngay ở dưới đây các bạn nhé!

Bộ sách Tổng hợp Bài mẫu IELTS Writing Sample 2023

Bộ sách Tổng hợp Bài mẫu IELTS Writing & Speaking Sample 2023

DOL IELTS Đình Lực dành tặng miễn phí bộ sách TỔNG HỢP BÀI MẪU IELTS WRITING & SPEAKING SAMPLE 2023. Tải về ngay ở dưới đây các bạn nhé!

Bộ sách Tổng hợp Bài mẫu IELTS Writing & Speaking Sample 2023

Đăng ký test đầu vào IELTS miễn phí và nhận tư vấn

Nhắn tin DOL qua Facebook

Click để nhắn tin với DOL qua fanpage chính

Gọi điện liên hệ

Liên hệ DOL qua hotline miễn phí: 1800 96 96 39

DOL có 15+ cơ sở tại TP.HCM, Hà Nội và Đà Nẵng

Click để xem địa chỉ chi tiết

promotion-background

The official IELTS by IDP app is here! Download it today.

  • Bài thi IELTS IELTS Academic IELTS General Training IELTS UKVI IELTS One Skill Retake Tìm hiểu bài thi IELTS Học Thuật là gì? Tìm địa điểm thi IELTS trên giấy Tìm địa điểm thi IELTS trên máy tính Đổi lịch thi hoặc hủy thi IELTS Hình thức thi IELTS Academic Thi IELTS trên giấy Thi IELTS trên máy tính Tìm hiểu bài thi IELTS Tổng Quát là gì? Tìm địa điểm thi IELTS trên giấy Tìm địa điểm thi IELTS trên máy tính Đổi lịch thi hoặc hủy thi IELTS Hình thức thi IELTS General Training Thi IELTS trên giấy Thi IELTS trên máy tính Tìm hiểu bài thi IELTS UKVI là gì? Tìm địa điểm thi IELTS trên giấy Tìm địa điểm thi IELTS trên máy tính Đổi lịch thi hoặc hủy thi IELTS Hình thức thi IELTS UKVI Thi IELTS trên giấy Thi IELTS trên máy tính TỔNG QUAN VỀ IELTS ONE SKILL RETAKE IELTS One Skill Retake là gì? Hướng dẫn đăng ký thi IELTS One Skill Retake TÌM HIỂU THÊM VỀ IELTS ONE SKILL RETAKE Các tổ chức chấp nhận IELTS One Skill Retake Cách sử dụng kết quả IELTS One Skill Retake Câu hỏi thường gặp IELTS One Skill Retake Đăng ký thi IELTS Lịch thi IELTS Lệ phí thi IELTS Chương trình ưu đãi IELTS Tìm địa điểm thi IELTS

Get your results

Check your provisional IELTS results online and do more.

  • Simplified Chinese

agree or disagree essay outline

Cách viết Agree or Disagree đạt điểm cao IELTS Writing Task 2

Dạng bài Agree or Disagree yêu cầu thí sinh thể hiện sự đồng ý/không đồng ý đối với ý kiến được nêu lên trong đề bài và giải thích cho sự lựa chọn của bạn.

Chủ đề bài viết

Dạng bài Agree or Disagree thường xuất hiện trong bài thi IELTS Writing Task 2 . Dạng bài này yêu cầu thí sinh thể hiện sự đồng ý/không đồng ý của mình đối với ý kiến được nêu lên trong đề bài và giải thích cho sự lựa chọn của bạn. Hãy cùng tham khảo cách viết Writing Task 2: Agree or Disagree với đề bài sau:

Smoking is a major cause of serious illness and death throughout the world today. In the interest of public health, governments should ban cigarettes and other tobacco products. Do you agree or disagree ?

>>>Xem thêm: Chi phí thi IELTS tại IDP

Bước 1: Đọc kỹ đề bài và xác định dạng bài

Điều đầu tiên thí sinh cần làm là đọc kĩ đề bài để xác định đây có đúng là dạng bài Agree or Disagree hay không. 

Dấu hiệu nhận biết dạng bài này là:

Key: Agree or Disagree 

Dạng câu hỏi khác: To what extent do you agree or disagree with this opinion? 

Ex: Increasing car use is contributing to global warming and having other undesirable effects on people’s health and well-being. To what extent do you agree or disagree with this statement?” 

Ở dạng đề này sẽ có có hai câu hỏi thông dụng: 

Câu hỏi 1 vế: Đưa ra 1 câu trần thuật và thí sinh lựa chọn đồng ý hay không đồng ý với ý kiến đó.

Ex: “In the interest of the public health, governments should ban cigarettes and other tobacco products” chính là câu trần thuật để thí sinh đưa ra ý kiến đồng ý hoặc không đồng ý. 

Câu hỏi 2 vế: Đưa ra 1 câu nêu ý kiến có 2 vế, lúc này, thí sinh có thể lựa chọn đồng ý/ không đồng ý với cả hai về hoặc chỉ đồng ý với 1 vế trong câu.  

Ex: “We cannot help everyone in the world that needs help, so we should be concerned with our own communities and countries. To what extent do you agree or disagree with this statement?

Being a celebrity, there are benefits as well as problems to be faced in society. To what extent do you agree with this view? 

For a long time art has been considered an essential part of all cultures in the world. However, nowadays people’s values have changed, and we tend to consider science, technology and business more important than arts. Do you agree or disagree? 

Computers are being used more and more in education and so there will soon be no role for teachers in education? To what extent do you agree or disagree?”

Xem thêm: Làm sao để hiểu đề bài luận trong Writing Task 2

Bước 2: Xây dựng outline

Sau khi đọc đề, bạn phải xác định quan điểm của bản thân là đồng ý hay không đồng ý với ý kiến trên. 

Lên Outline khái quát Bài viết gồm 3 phần chính : 

Introduction: Giới thiệu vấn đề, khẳng định mình đồng ý hay không đồng ý.  

Body 1,2,3: Đưa ra các luận điểm cho ý kiến của bản thân, bổ sung các luận cứ chứng minh luận điểm đó, nêu ví dụ và kết luận. Mỗi đoạn của phần thân bài sẽ bao gồm:

Topic sentence (1 câu)

Explain topic sentence (2-3 câu)

Example (1 câu)

Conclusion: Khẳng định lại vấn đề và quan điểm của bản thân.

IELTS CD Booklet banner

DOWNLOAD NGAY

Bước 3: Triển khai chi tiết từng phần

Introduction.

Để có thể triển khai chi tiết cho đoạn mở bài, thí sinh cần làm theo ba bước sau đây:

B1: Xác định câu chủ đề và câu hỏi 

Phân tích trên đề bài chính:

Câu chủ đề: In the interest of the public health, governments should ban cigarettes and other tobacco products. 

Câu hỏi: Agree or disagree? 

B2: Paraphrase lại câu chủ đề

Ở bước này, thí sinh không nên chép lại hoàn toàn cầu chủ đề, thay vào đó hãy paraphrase lại bằng cách sử dụng chủ ngữ giả định và từ đồng nghĩa. 

Ex: It is believed that governments should pass a law to prohibit types of tobacco products, including cigarettes for a healthy community. 

B3: Trả lời câu hỏi 

Ở bước này, bạn sẽ nêu lên quan điểm của mình đối với vấn đề. Quan điểm của bạn sẽ được thể hiện ở ba mức độ:

Hoàn toàn đồng ý: I totally/completely/strongly agree with this idea. 

Hoàn toàn không đồng ý: In my opinion, I disagree with this view. 

Đồng ý/ không đồng ý một phần: From my point of view, I partly agree/disagree with this view. 

Lưu ý: 

Với câu hỏi 1 vế, bạn chỉ cần đồng ý/ không. Có thể sử dụng các cấu trúc để nêu ra quan điểm bằng cách câu sau: 

In my opinion, I completely agree/disagree. 

According to…/ Following to… (Theo như), so I completely agree/disagree

In fact/ Factually/In reality/It is true that  (Thực tế là) … , so I completely agree/disagree

Với câu hỏi 2 vế, nếu bạn đồng ý/ không với cả 2 vế thì có thể dùng như trên. Nếu bạn chỉ đồng ý 1 vế

Although I agree that [quan điểm về vế thứ 1], I still believe [quan điểm về vế thứ hai] “

Ex: Although I agree that technology, business play a vital role nowadays, I still believe that science, technology and arts are equally influential in a community's life. 

Một số từ trình bày ý kiến bạn cần biết để làm tốt phần mở bài:

In my opinion 

I strongly agree/disagree with the idea that…

My opinion is that…

In my view,...

From my point of view, 

From my perspective,...

As far as I am concerned,...

It seems to me that…

I am of the opinion that…

Xem thêm: Cách viết phần mở bài hay trong IELTS Writing Task 2

Supporting Paragraphs

Ở phần này, bạn sẽ phát triển 2 câu luận điểm thành đoạn văn bằng cách đưa ra dẫn chứng, ví dụ và kết luận. 

Luận điểm thứ nhất, gồm 4-5 câu 

Câu 1: Đưa ra luận điểm.

Câu 2-3: Giải thích lý do. 

Câu 4: Ví dụ => Cấu trúc As an example,…/For example,…/ For instance,… (Ví dụ như), Specifically,…(Cụ thể là), To illustrate… (Để minh họa).

Câu 5:  Tóm tắt lại ý tưởng của cả đoạn.

Một số cấu trúc câu triển khai luận điểm:

Firstly,… → Secondly,… → Thirdly,… →… (Thứ nhất, thứ hai, thứ ba,…)

In the first place,… → In the second place,… → In the third place,… →… (Ở nơi thứ nhất, ở nơi thứ hai, ở nơi thứ ba,…)

To begin with… → Next… → Finally,… (Bắt đầu với, tiếp theo là, cuối cùng là

Ex: Firstly, cigarettes and tobacco goods always provide more adverse effects to people who are smoking; Types of tobacco products have a high nicotine level. When people smoke, these enormous toxic nicotine amounts can enter upon people’s bodies and attack viscera, especially the lung. For an illustration, a recent report from WHO said that more than 70% of lung cancer comes from smoking around the world and more than 80% of the population smoke around the globe. That is why these tobacco products should be banned by the authorities. 

Luận điểm thứ hai, gồm 4-5 câu và tương tự như trên

Nếu là dạng câu hỏi 2 vế thì Body 1-2 chính là luận điểm đồng ý/ không đồng ý của mỗi vế. Khi đó bạn nên sử dụng cấu trúc đối lập như:

On the one hand,… → On the other hand,… (Một mặt thì, mặt khác thì)

In contrast,…/ On the contrary,… (Đối lập với)

However,…/ Nevertheless,… (tuy nhiên)

Meanwhile,… (Trong khi đó)

Ex: However, the prohibition of tobacco goods can lead to a crime explosion. Nicotine is an addictive substance, so it makes people feel out of control. If they can not get it, they can manage to smuggle tobacco goods. That is the way to increase social crime percentages. According to WHO, 22.3% of the global population used tobacco in 2020. Make your own imaginations, what will happen if all of them create illegal things. 

Conclusion: Gồm 1 câu tóm lại ý kiến của bản thân

Một số cụm từ phổ biến dùng cho phần kết luận: 

In conclusion (kết luận lại thì)

To summarize/ In sum /In closing (tóm lại là)

To conclude (để kết luận)

At last/ Finally (cuối cùng)

In brief /On the whole/Overall  (nói chung)

After all (sau tất cả)

Ex: In conclusion, the proportion of diseases and death can reduce dramatically as long as people stop smoking. Hence, the government should make a law to bring this into effect. 

Xem thêm: Cách viết đoạn kết luận tốt trong Bài thi IELTS Writing Task 2

Bước 4: Ghép lại được bài viết hoàn chỉnh và kiểm tra lại toàn bài

Ở bước này, bạn sẽ ghép tất cả các phần đã viết và đọc lại toàn bài để đảm bảo tính logic, mạch lạc cũng như kiểm tra các lỗi chính tả.

Lưu ý các lỗi sai thường gặp

Xác định quan điểm không rõ ràng: Luận điểm của bạn phải rõ ràng, bám sát câu khẳng định của bạn ở phần mở bài:

Phát triển luận điểm chưa đủ tính thuyết phục: Bạn không nên chỉ diễn giải các câu lý thuyết suông, thay vào đó, để bài viết có tính thuyết phục hơn, hãy đưa ra các dẫn chứng, ví dụ từ các nguồn uy tín. 

Lỗi sai ngữ pháp, chính tả: Bạn cần củng cố kiến thức ngữ pháp, đặc biệt là cách sử dụng các cấu trúc câu. Bên cạnh đó, từ vựng của bạn cũng cần liên quan mật thiết đến chủ đề và tránh tuyệt đối các lỗi chính tả.  

Xem thêm: 

Cách cải thiện vốn từ và chính tả của bạn

Cách chỉnh sửa bài viết IELTS để tránh lỗi sai

Bài mẫu IELTS Writing task 2 agree or disagree

Để thành thạo hơn cách viết Writing Task 2 dạng bài Agree or Disagree, bạn có thể thực hành bằng các phân tích các bài mẫu để rút ra cách thức thực hiện và rút kinh nghiệm cho bản thân. 

Ex: Scientific research should be carried out and controlled by governments rather than private companies. Do you agree or disagree?

Scientific research plays an important role in the development of humanity. Some people think that the responsibility for conducting and managing it should be taken by governments, and not by private companies. Personally, I believe that both governments and private businesses should have the right and duty to carry out research.

Ở phần mở bài, tác giả bài viết đã dành một câu để khẳng định vai trò của Scientific, chủ thể chính trong đề bài. 

Câu chủ đề: “Some people think that the responsibility for conducting and managing it should be taken by governments, and not by private companies.”

Câu này được paraphrase lại từ câu chủ đề của đề bài. 

Câu trả lời: “Personally, I believe that both governments and private businesses should have the right and duty to carry out research.”

Tác giả khẳng định rằng cả “government” và “private businesses” đều có quyền và nghĩa vụ tiến hành các nghiên cứu khoa học. 

I believe that governments should play the leading role in performing research due to a number of reasons.

Firstly, scientific research is the key to finding solutions to the pressing concerns of society. Thorough medical research, for example, needs to be conducted by governments to develop vaccines against potential epidemics. Research into renewable energies is also crucial to deal with global warming. These kinds of research require a great deal of spending, which can only be afforded by the government. In addition, scientific studies controlled by governments will ensure that reliable and ethical methods are used, and minimize the risk of these studies

being used for criminal activity. For instance, studies related to nuclear power or weapons must be carried out or supervised by governmental organizations to protect a country from terrorism or other warfare.

Topic sentence: Firstly, scientific research is the key to finding solutions to the pressing concerns of society.

Explain topic sentence:  

Thorough medical research, for example, needs to be conducted by governments to develop vaccines against potential epidemics.

In addition, scientific studies controlled by governments will ensure that reliable and ethical methods are used, and minimize the risk of these studies being used for criminal activity.

Example: 

For instance, studies related to nuclear power or weapons must be carried out or supervised by governmental organizations to protect a country from terrorism or other warfare.

However, private companies can do research which is not carried out by governments. In fact, many useful scientific discoveries have been made by private institutions. To illustrate, the invention of the iPhone by Apple, a corporation in the US, began the era of smartphones and facilitated modern forms of interaction between people all over the world. Furthermore, encouraging profit-driven companies to conduct scientific investigations will help them develop their own products and create competition in the marketplace. Society will therefore benefit from new inventions and better products.

Topic sentence: However, private companies can do research which is not carried out by governments.

Explain topic sentence:

In fact, many useful scientific discoveries have been made by private institutions. 

Furthermore, encouraging profit-driven companies to conduct scientific investigations will help them develop their own products and create competition in the marketplace.

To illustrate, the invention of the iPhone by Apple, a corporation in the US, began the era of smartphones and facilitated modern forms of interaction between people all over the world.

In conclusion, although governments should control and carry out major scientific studies, private companies should not be deprived of the right to do research.

Ở phần này, bạn đã tái khẳng định quan điểm cá nhân rằng cả chính phủ và các doanh nghiệp tư nhân đều có quyền triển khai các nghiên cứu. Một bên không nên ngăn cản và một bên cần được cấp quyền. 

Cách viết dạng Problem and Solution trong Writing Task 2: chi tiết 4 bước

Cách viết IELTS Writing task 2 dạng Opinion chi tiết giúp đạt điểm cao

Hướng dẫn cách viết IELTS Writing Task 2 dành cho mọi dạng bài

Hướng dẫn cách viết Argumentative Essay trong IELTS Writing

Tựu trung, IELTS Writing Task 2 là một phần thi khá khó với hầu hết các thí sinh. Tuy nhiên, khó không có nghĩa là không thể chinh phục. Bạn hoàn toàn có thể làm tốt phần này nếu có sự chuẩn bị và đầu tư kỹ lưỡng về mặt kiến thức và kỹ năng, trong đó, phân tích các bài mẫu, tự mình lên outline và thực hành viết bài với tần suất dày đặc là phương pháp quan trọng nhất.

Về bài viết

Published on February 03, 2023

Quỳnh Khanh

Tôi là Quỳnh Khanh - Content Writer có hơn 5 năm về lĩnh vực giáo dục

Chia sẻ bài viết

Các chủ điểm ngữ pháp IELTS mà ai cũng nên biết

Phân biệt thì Hiện tại hoàn thành và Hiện tại hoàn thành tiếp diễn

Cách trình bày ngày tháng chính xác

Cách viết số trong bài thi IELTS

IELTS 5.5 là cao hay thấp? Cách đạt band 5.5 trong thời gian ngắn nhất

Tiêu chí chấm Grammatical Range và Accuracy trong IELTS Writing và Speaking

IELTS Writing band descriptors - Tiêu chí chấm IELTS Writing

Top 10 sách ngữ pháp IELTS từ cơ bản đến nâng cao cho mọi trình độ

Trọn bộ 14+ cuốn sách luyện thi IELTS cho người mới bắt đầu

[Hướng dẫn] Xây dựng lộ trình học IELTS 5.0 hiệu quả cho người mới bắt đầu

  • Hình thức thi IELTS
  • Các tổ chức công nhận IELTS
  • Tin tức và bài viết cập nhật
  • Sự kiện IELTS
  • Hình thức thi nào phù hợp với tôi?
  • Chương trình ưu đãi IELTS
  • Đối tác IDP IELTS tại Việt Nam
  • Câu chuyện thành công
  • IELTS - Tấm vé vàng để đi du học
  • Liên hệ IDP IELTS
  • Thi IELTS trên máy tính
  • Thi IELTS trên giấy
  • IELTS Học thuật
  • IELTS Tổng quát
  • IELTS UKVI Học thuật
  • IELTS UKVI Tổng quát
  • IELTS UKVI Life Skills
  • IELTS One Skill Retake
  • Đăng ký thi IELTS
  • Lịch thi IELTS
  • Lệ phí thi IELTS
  • Tìm địa điểm thi IELTS
  • Câu hỏi thường gặp về IELTS
  • Bài kiểm tra IELTS Familiarisation
  • Ứng dụng IELTS by IDP
  • IELTS Masterclass
  • Tải các mẫu đơn
  • Cam-pu-chia
  • Cộng hòa Síp
  • Netherlands
  • New Caledonia
  • New Zealand
  • Papua New Guinea
  • Philippines
  • Ả Rập Xê Út
  • Quần đảo Solomon
  • Tây Ban Nha
  • Quần đảo Ellice
  • Các Tiểu Vương quốc Ả Rập Thống nhất
  • Liên hệ với chúng tôi
  • Thông báo pháp lý
  • Chính sách quyền riêng tư
  • Chính sách cookie
  • Bản quyền 2024 IDP IELTS

COMMENTS

  1. IELTS agree or disagree essay

    This is a band 9 guide to writing agree/disagree essays in IELTS Writing. Agree or disagree essay questions are very common for IELTS Writing task 2.This type of questions asks you to say whether you agree or disagree with a given statement and justify your opinion.. In this lesson you will see IELTS writing task 2 sample question + model answer and learn

  2. How to Write "Agree or Disagree" Essays for IELTS

    As such, here are two possibilities for structuring your "agree or disagree" essay: Introduction. Introduce the topic. State your position ( essay outline) Body paragraph #1. Main argument #1. Support with explanation and example. Body paragraph #2. Main argument #2.

  3. IELTS Writing Task 2: How to Structure an 'Agree or Disagree' Essay

    Paragraph 2 (body 1) Explain and support your first reason why rail is more important. Paragraph 3 (body 2) Explain and support your second reason why rail is more important. Paragraph 4 (conclusion) Re-state your overall opinion (rail is more important) and summarise your main reasons. 2.

  4. IELTS Agree or Disagree Essay

    Structure of IELTS Opinion Essay. Paragraph 1: Introduction. Paraphrase the Essay Topic. Thesis Statement. Paragraph 2: Supporting Paragraph #1. Topic Sentence. Support (Example or Experience) Explanation. Paragraph 3: Supporting Paragraph #2.

  5. Ultimate IELTS Band 7+ Structure for Agree or Disagree Essay

    Provide a brief overview of the main points you will discuss in the essay. Body Paragraph 1: Start with a strong topic sentence that presents the main reason supporting your agreement or disagreement. Provide supporting evidence, examples, or data to justify your position. Elaborate on the implications or consequences of your viewpoint.

  6. Agree or Disagree Essay- Complete 2023 IELTS Lesson

    Agree or Disagree Essays: IELTS Writing Task 2 Lesson. This lesson will teach you how to write 'agree or disagree' or 'opinion' IELTS task 2 essays that could score a Band 7, 8 or 9. ... Sentence 3- Outline Statement. Paragraph 2- Supporting Paragraph 1. Sentence 1- Topic Sentence. Sentence 2/3- Explain Topic Sentence.

  7. IELTS Opinion Essays

    IELTS opinion essays, also known as 'agree or disagree' essays, come up frequently in the writing exam. In this lesson, I'm going to show you how to plan and write them step-by-step. Here's what we'll be covering: 3 Common mistakes. Essay structure. How to plan. How to write an introduction. How to write main body paragraphs.

  8. A Simple Formula for Organizing Agree/Disagree Essays

    Sentence 1: States the premise that the essay will address (you can simply rephrase the words in the task). Today it is a popular belief that students attending high school should volunteer in the community as part of their schooling. Sentence 2: States your opinion in the matter. I strongly agree with this notion.

  9. IELTS Agree/Disagree Essay: How to Write, Structure, Tips, Sample

    Full Disagreement: Dedicate the first paragraph to clearly outline your counter-arguments against the statement. The second paragraph can strengthen your stance by introducing further dissenting points. ... Sample IELTS Agree/Disagree Essays: Here are some sample IELTS agree/disagree essays that you can refer to whilst attempting the IELTS ...

  10. Mastering the IELTS Agree Disagree Essay: Tips for Success

    The agree or disagree essay questions are the most commonly asked in the IELTS examination. They are also called as argumentative essays. In this type of essays, you are asked to give your opinion, whether you agree or disagree on the particular sentence that is given. You are given 40 minutes to complete the 250-word essay.

  11. IELTS Task 2 Question Types: Opinion Essays (Agree or Disagree)

    Here I have collected actual IELTS opinion essays (agree or disagree question) from the last several years - enjoy learning about this task type! Enjoy and consider signing up for my Patreon Ebooks here. Dave. IELTS Task 2 Question Types: Opinion Essays (Agree or Disagree) Individual greed and selfishness have been the basis of modern society.

  12. How To Write An Agree Or Disagree Essay?

    In this post, we will be focussing on how to write an agree/disagree/opinion essay. One of the first things you should do is read the marking criteria to see what the examiners expect. This is really important, as you need to know what they are looking for in the band 7+ boxes. ... Outline the specific topics that will be discussed in the main ...

  13. How to Start an Agreement Essay

    Thus, here is how you should start an agreement essay: 1 st sentence. Introduce the topic. 2 nd sentence (optional) Further explain the topic. 3 rd sentence. Present your opinion. You often hear people say that you should paraphrase the question.

  14. IELTS Writing Task 2 Essay Structures + Band 9 Essays

    The five most common IELTS Writing Task 2 questions are: Opinion (Agree or Disagree) Advantages and Disadvantages. Problem and Solution. Discussion (Discuss both views) Two-part Question. Below I will outline examples and a structure approved by experienced IELTS teachers and examiners for each type of question.

  15. How to organize agree/disagree essays on your IELTS exam

    Sentence 1 : States the premise that the essay will address (you can simply rephrase the words in the task). Today it is a popular belief that students attending high school should volunteer in the community as part of their schooling. Sentence 2 : States your opinion on the matter. I strongly agree with this notion.

  16. How to write an agree/disagree essay for…

    You partially agree - First paragraph: reasons why you agree. 2nd paragraph: reasons why you disagree. Remember: it's much better to have few well-developed ideas than a lot of poorly developed ones, so when you write the paragraphs make sure to give reasons, examples and details. All these must be relevant to the reason you agree/disagree.

  17. An opinion essay

    Introduce your essay by restating the question in your own words. If the essay asks you to what extent do you agree?, make your opinion clear throughout. You can either agree, partially agree or disagree with the statement, explaining and justifying your opinion. The structure should be: Introduction.

  18. Opinion essay

    An "agree or disagree" opinion essay is a type of essay where you are presented with a statement or an opinion and must express your agreement or disagreement with it. The key to writing an effective "agree or disagree" essay is to present a clear opinion and support it with well-developed arguments. It is important to provide evidence ...

  19. IELTS ESSAY TEMPLATE-agree

    A -. INTRODUCTION. To agree or disagree with the statement that (summary of the question) is an important issue. Putting the discussion in a wider context, ( a few words about the subject) has always been debatable. Even though some people think that (the idea which you dont like) , I wholeheartedly believe that ( the idea which you like).

  20. Education

    Education - IELTS Agree/Disagree Essay Sample. In this post, we will look at an agree/disagree essay example from the IELTS writing task 2 test. Students often ask if the questions are repeated year after year and the answer is no, but the topics are. There are so many questions written each year, you may find your practice answering various ...

  21. How to write a Thesis statement in IELTS essays. IELTS writing task 2

    For each essay type this is what should be in the Thesis statement: 1. Opinion essays: write 2 reasons for your opinion. 2. Advantage disadvantage essays: state the advantage and the disadvantage, 3. Problem solution essays: briefly state 1 or 2 problems and possible solutions. 4.

  22. Cách làm dạng IELTS Writing Task 2 Agree or Disagree kèm mẫu

    Cách viết IELTS Writing Task 2 dạng Agree or Disagree chi tiết. Dạng bài viết Agree-Disagree Essay (hay còn gọi là Opinion Essay) là 1 trong 4 dạng câu hỏi chính mà bạn sẽ gặp khi thi ielts writing task 2 agree or disagree.Mật độ của dạng đề này tương đối cao nên bạn cần có một phương pháp làm bài cụ thể cho dạng đề này ...

  23. Cách viết Agree or Disagree đạt điểm cao IELTS Writing Task 2

    Cách viết Agree or Disagree đạt điểm cao IELTS Writing Task 2. Dạng bài Agree or Disagree yêu cầu thí sinh thể hiện sự đồng ý/không đồng ý đối với ý kiến được nêu lên trong đề bài và giải thích cho sự lựa chọn của bạn. Dạng bài Agree or Disagree thường xuất hiện trong bài ...